Moychay Menghai Gongting and Bada shou pu'er comparison

Multiply Page 3


Psychiatry.Multiply.Com

Blog EntrySep 27, '08 12:56 PM
for you
Station
45 yr male, depressed on fluoxetine, improved mood but unwilling to take it. Explore the reasons for this (sexual side-effects, but he doesn't give anything away unless you ask directly)
Linked Station
Discuss with wife, paying attention to her concerns (she can be quite difficult)
Similar link in the past
A 35 year old woman who is the wife of one of your patients has arranged to see you. Three months ago her husband was diagnosed with depression and started on fluoxetine. Now his symptoms have partially responded to a 40 mg dose. However he has sexual side effects from the medication. The patient himself was too embarrassed to come to the appointment, but has given you permission to discuss matters with his wife. Elicit and address her concerns.

This is a combined history taking and information giving station. Trickcyclists classify it under communication skills but have no detail about it as yet. My approach would be:


Introduction:
  • I gather that you have been depressed for a while and taking medicaion for depression which has helped you but you are reluctant to continue that medication. I am here to discuss this issue with you. Can I ask you what is really bothering you and why you don't want to take this medication. If it is the partner of the  patient, ask her whether the person himself knows that we are talking about his problems and is he happy about that.
  • I appreciate that this might be a very uncomfortable thing to talk about, but would you mind telling me a little bit more about your reasons for thinking that it's the medication that has lead to this problem.
  • Some people find that when they are intimate with their partners, they cannot perform upto their expectations, did you have  the  similar situation in your case?


    H.O.P.I.
  • Ask for how long he is taking these tablets and in what dosage.
  • What are the main side effects really bothering you.
  • Are they related to your sexual performance.
  • Ask about sexual performance
  • Ask about morning erections 
  • When did he start having this symptom.
  • Is this before medication or after?
  • Patient responds.


  • Differential Diagnosis:
  • Ask about his mood whether he is depressed or not.
  • Any such problem in the past.
  • Ask if he is suffering from any physical illness such as diabetes.
  • Ask questions to r/o erectile dysfunction caused by anxiety/Depression, HTN, DM, ETOH
  • Also enquire about his pre-morbid sexual function and his current relationship.
  • Ask if he drinks and in how much amount and how frequently.
  • Ask about substance misuse
  • Ask about relationship with the partner


    Discussion:
  • Tell that these may be due to the tablets he is taking.
  • Has he taken any other antidepressant before and for how long and in what dose.
  • One or two.
  • Tell about reduction in dose
  • Tell about alternate treatments such as mirtazapine and you would discuss with your consultant and other team members and then have a detailed chat with you.
  • Tell him that meanwhile it is important to take and continue on fluoxetine
  • Ask him whether he knows about risk of relapse.
  • Tell him that the problem caused by relapse of depression and that if he continues with fluoxetine,  for a temporary period and we would change him to an alternate treatment after discussion with the consultant.
  • Offer him if he wants leaflets on alternate medications.
  • Reassure about your help and tell him about your availability.
  • Arrange a follow up appointment in 2-3 weeks.
  • Suggest doing blood tests,encourage him in the mean time to take his medication (as they had helped control the depression)
  • Ask him for any questions.

  • Linked Station

    Discuss with wife, paying attention to her concerns (she can be quite difficult)
    • I gather that you want to discuss about your husband. I have asked him and he is happy that I discuss this with you.
    • Can you tell me what is bothering you. 
    • Tell her about history of presenting illness whether it started before antidepressant or after.
    • Address her concerns
    • Tell her about past psych history, past medical history, any substance misuse etc. if there or not
    • Empathise
    • Tell about medication
    • Apologise if side effect not discussed earlier on
    • Tell about reversible nature of the problem
    • Tell her about reduction of dose
    • Tell about alternative solutions
    • Reassure her (no false reassurance)
    • Tell her about blood tests
    • Offer her leaflets
    • Offer her appointment


     
    Blog EntrySep 26, '08 2:19 PM
    for Kamran's contacts
    21 yr college boy, was in car accident with father when his dad was killed, develops diplopia and loss of vision at father's funeral. Take Hx, fundoscopy and other relevant physical exam

    Link- speak to mum, address her fears and anxieties

    ____________________________________________________________________________
    Station A
    Good aftrenoon, I am Dr. Shaikh and I am one of the psychiatrists. I understand from the letter that your father has passed away. I am very sorry about that.
    • Show empathy
    • Express condolence
    Loss of vision
    • since when
    • How did it happen
    • What you can see and what not
    • Able to see anything?
    • Able to feel presence of light in the room?
    • One eye/both eye?
    • Did you get injured?
    • Medical History such as diabetes or hypertension?
    • How bothered are you about this?
    • Any ideas how your vision can be improved?
    DIPLOPIA
    • How?
    • Where?
    • When?
    • Which eye?
    • What does it mean to him?
    • All the time or some time?
    • Bothered or not?
    • Any ideas how it can be improved?
    OTHER HISTORY
    • History of bereavement
    • How was his relation to his father
    • How did he die?
    • How does he feel now?
    • Any suicidal ideations
    FUNDOSCOPY + Physical Exam
    EYE Examination
    • Check visual acuity
    • Check pupillary reaction
    • Simple inspection with naked eye and torch 
    • Check eye movements
    FUNDOSCOPY
    • Right hand for right eye and left for left
    • Explain procedure (to look at the back of your eye) and that it won't harm him
    • Check if he wears any glasses; set to 0 if does not wear glasses
    • Ask him to look straight on one object
    • Comment on what you are seeing Fundus, Optic disc, arteries, any haemorrhages, soft exudates, hard exudates etc.
    OTHER EXAMINATION
    • Blood pressure
    • Carotid bruit
    • Heart sounds if time allows
    _________________________________________________________________________
    Station B
    • Speak to mother
    • First ask about her concerns
    • Tell her about onset and nature of his symptoms as gathered from history
    • Allay her fears and anxieties
    • Tell her about physical examination
    • Allay her fears and anxieties
    • Tell her that stress can produce symptoms like that
    • Tell her that what psychological factors may have leaded to it?
    • Tell that if you have suspected any eye disease, you would have referred him to ophthalmology without any delay
    • Take into her confidence about tretatment
    • Self remission
    • Psychotherapy
    • Instill hope
    • Thank her
    • Offer leaflets about conversion disorder
    • Offer follow up appointment
    Blog EntrySep 25, '08 4:12 PM
    for Kamran's contacts
    1) Louise is a 15yr girl, who has taken an overdose of 80 tablets of paracetamol and arrived in the A&E. Medical team has asked you to see her. Take relavent history with special emphasis on aetiological factors and do a risk assessment. She was assessed 3 days ago by your consultant who found that she does not suffer from a depressive illness.

    Link- discuss the case with consultant. Formulate your assessment and management plan.


    _____________________________________________________

    Checklist
    1. Circumstances of overdose and current intent
    2. Past attempts
    3. Family dynamics
    4. Personal History - bullying abuse etc.
    5. Substance misuse
    6. Personality traits - borderline
    _____________________________________________________________________________

    LINK - Discuss with the consultant
    1. Present demographic data
    Name, age, sex, living with whom, how many siblings, where assessed, referred by whom, why referred, brought by whom,  medical status.
    2. Risk Assessment and Details of Overdose
    3. Past attempts
    4. Family dynamics
    5. Personal History - bullying abuse etc.
    6. Substance misuse
    7. Personality traits - borderline
    8. Management plan
    Depends on history
    I shall consider admission if any of the following factor is present in the history:
    • Child is too young (not in this case)
    • Serious attempt and method
    • Toxic environment
    • If the child ran away from home before attempting suicide
    • Comorbid Psychiatric disorder (Axis I ruled out in this case)
    I shall consider Outpatient management if the above factors are not present.
    That would include involving family , commencing treatment for psychiatric disorder (not in this case) and a regular follow up.
    In the long term I would take measure for secondary prevention of these attempts which may include:
    • Keeping the child away from any potential weapons or methods of suicide
    • Parental education and training

    Blog EntrySep 18, '08 4:17 PM
    for Kamran's contacts

    Name of the candidate:
    AREAS OF CONCERN
    Shade lozenge if concerned
    1. Questioning Style e.g. use of approximate mix of open & closed questions
    5
    2. Listening and responding appropriately to interview/discussant
    5
    3. Management of interview/examination including empathic responses
    5
    4. Lack of appropriate focus on the required task
    5
    5. Fluency of interview/examination/discussion
    5
    6. Professionalism including but not limited to harmful interaction; failure to respect individual’s rights; ethical behaviour etc.
    5
    7. Appropriate choice of avenues of enquiry, tests or examination including significant omissions
    5
    8. Range and/or depth of history explored
    5
    9. Range and/or depth of psychopathology explored (signs elicited at examination)
    5
    10. Range and/or depth of risk explored
    5
    A = Pass
    B = Borderline Pass
    C = Borderline Fail
    D = Fail
    PASS:
    0/1 lozenge filled = A = Pass
    2 lozenges filled = B = Borderline Pass
    Must get 8/10 to pass a station
    FAIL:
    3/4 lozenges filled = C = Borderline Fail
    5/more lozenges filled = D = Fail
    Blog EntrySep 3, '08 9:37 AM
    for you
    Question 1:
    Abreaction
    Can abreaction still be used in the treatement of dissociative disorders?
    Ideal designs:
    RCT
    Open Trial
    Case Series
    Case Study
    Question 2:
    Blog EntryAug 26, '08 2:54 PM
    for everyone
    • 75 % of patients with schizophrenia will become non-compliant after 18 months. (CATIE study).
    • Nocturnal enuresis cannot be diagnosed in a 4 year old child; the minimum age for this diagnosis is 5 years.
    • If you find white matter hyperintensities in a CT scan brain of an elderly man, he is more likely to get late onset depression than late onset schizophrenia.
    • Adolescents respond better to antidepressants than do children.
    • Although mostly due to substance  misuse or functional in nature, Othello syndrome may also be related to cerebral tumour.
    • Increased maternal age is a risk factor for Down's syndrome, but not for post-natal depression.
    • Bizarre postures are more common after ingestion of PCP than are visual hallucinations.
    • Heritability of schizophrenia and bipolar affective disorder is about 80-85%.

    Blog EntryAug 23, '08 11:09 AM
    for everyone
    Q 1. A sociocultural explanation rather than a biological one in learning disabilities is most accounted for by:
    1. An even spread of Learning Disability across different socio-economic groups of the population.
    2. Moderate Learning Disability
    3. Learning Disability in other members of the family
    4. Facial dysmorphologies
    5. Problems with adaptive functioning 
    Answer: 
    5. Problems with adaptive functioning

    http://64.233.183.104/search?q=cache:2qOlrDexzL4J:www.dmacc.edu/Instructors/acstevens/241ppts/comer5e_lecture_Ch03.ppt+socio-cultural+model+Mental+Retardation&hl=en&ct=clnk&cd=3&gl=pk

    Questions 2-8
    For question 2 – 8 Please read the following article from the 
    BJPsych 2007 190 : 129-134View Table 

    The table they gave us had the following factors as most significant (in order of Odds Ratio and Confidence Intervals)
    1. Previous falls
    2. Inhaled corticosteroids
    3. Injected corticosteroids
    4. Schizophrenia diagnosis
    The article was:
    The British Journal of Psychiatry (2007) 190: 129-134.
     BJP ArticleResults:
    What followed was this table : 
    http://bjp.rcpsych.org/cgi/content/full/190/2/129/TBL2 

    EXCEPT – all they showed us was schizophrenia, inhaled corticosteroids, injected corticosteroids, a few more (NO DRUGS!) , and they added:

    Previous Falls OR 3.2 95% CI 3-5 (This was the most significant result on the table!)

    All of these results are from the univariate analysis.






    Q. 02: What type of study is this?
    1. Case Control
    2. Cohort
    3. RCT
    4. Qualitative
    5. Cross sectional Survey
    1. Case Control 


    Q. 03:: What was the most significant predictor of hip fractures?
    1. Previous falls
    2. Schizophrenia
    3. Alcohol use
    4. Smoking
    5. Weight 
    5. Weight 

    Q 04: Which of the following predicts more hip fractures:
    1. Schizophrenia
    2. Inhaled corticosteroids
    3. Injected corticosteroids
    4. Smoking 5. Weight 
    5. Weight 

    Q 5: What can we not deduce from this study?

    1. Antipsychotics cause falls.
    2. Corticosteroids are significantly associated with hip fractures.
    3. A diagnosis of schizophrenia is significantly associated with hip fractures. 

    4. Cerebrovascular disease is significantly associated with hip fractures. 5. Hormone Replacement Therapy is protective against hip fractures3. A diagnosis of schizophrenia is significantly associated with hip fractures. 




    Q 6: According to this study, the next best step for someone who has had a fall would be to:

    1. Do a thorough falls risk assessment
    2. Check for schizophrenia
    3. Check alcohol use
    4. Check weight
    4. Check weight 



    Q 7: Which statement is the least true:
    1. The General Practice Research Database sample is random
    2. The practices are likely to have computerised data
    3. Three more true answers
    2. The practices are likely to have computerised data 
    The general practice Database was ranom though there was some selection bias toward large practices.


    For Q 08 and 09, please read the following précis:

    There is a well established depression scale (A) with 100 items and a scale (B) with 12 items. Scale B only takes 10 minutes to administer. Both scales A and B were administered to 100 depressed in-patients and 100 controls.

    Spearman’s correlation r = 0.8 p<0.01. High scores on A correlated with high scores on B

    Q 08: Which of the following is true:
    1. A and B are highly LINEARLY correlated
    2. A is a good screening tool
    3. B is a good measure of depression.
    4. B can replace A
    5. A and B show good inter rater reliability
    1. A and B are highly LINEARLY correlated
    Spearman's is near enough to 1( i.e. 0.8 )and p-value is less than 0.01 hence there is a strong correlation. The range of all coefficients is from -1 to +1. 


    Q 09: Which of the following is true:

    1.Because A is longer than B it must be better
    2. A is more valid than B
    3. B is more valid than A
    4. B is more reliable than A
    5. We have not been given enough information to assess reliability 

    5. We have not been given enough information to assess reliability 

    Learning Disability 

    Q 10. You are counselling a mother who has one child with autism and is expecting her second. She wishes to know the risk of autism in this second child. You say:
    1. 1 – 10%
    2. 10 – 20%
    3. 30 -40%
    4. 40 – 50%
    5. 50-60%
    1. 1-10% 
    ASD has a strong genetic component, but it's not as simple as you might imagine. In general, the likelihood of having a child with ASD is about 1 in 166. Once a couple have had one child with ASD, their risk of recurrence rises to somewhere between 1 in 50 and 1 in 12.Reference:http://autism.about.com/od/medicalissuesandautism/f/geneticsfaq.htm


    Q11. What increases the risk of Alzheimer’s disease in a child with Down’s syndrome? Answer: A family history of Alzheimer’s disease. 



    Q12. In Rett’s syndrome which is incorrect: 
    1. The child develops normally until 18 months. This is false and thus the desired answer. There are developmental delays before 18 months too, though symptoms become more characteristic after 18 months. 2. Mental deterioration precedes motor deterioration. This is also true. Although motor "delay" can occur before onset of mental symptoms, but motor "deterioration" per se occurs after mental deterioration.
    3. 40-50% show self-injurious behaviour This is true. According to one study 49% of patients self harm. 
    http://www.emedicine.com/PED/topic2653.htmhttp://www.ncbi.nlm.nih.gov/pubmed/8335149 http://books.google.com.pk/books?id=Q6xkh3QdyGEC&pg=PA29&lpg=PA29&dq=Rett+'s&source=web&ots=tAq_-cR5jq&sig=Qml4nS_eYoBddZu4scvlJTwY3Dw&hl=en#PPA30,M1


    Q13. 
    In LD which are true: Answer:1. Carbemazepine is associated with hyponatremia.Maudsley Prescribing Guidelines. 

    Q14. In relation to Learning Disability which of the following statements is true:
    1. Lamotrigine is a mood stabiliser. FMood stabilizer otherwise but not used as mood stabilizer in population with LD
    2. Propanolol is a good anti-manic agent FalsePropranolol is used for aggression occasionally but not as anti-manic
    3. Donepezil is of proven benefit for dementia associated with Down’s syndrome TRUE Ref. Seminars in Learning Disability

    Psychotherapy 

    Q15.

    During therapy a patient suddenly blurts out near the end of the session “I am abusing my children” before quickly shifting the topic to other things. It is almost the end of the session and the appropriate thing for you to do is:
    1. Say “What do you mean by abusing?”
    2. Keep your boundaries. End the session on time (ignoring what she has just said) and wait until the next session.
    3. Reassure her that everything said in therapy is confidential.
    4. Tell her that you have to report her to the authorities.

    1. Say “What do you mean by abusing?”

    This is my guess as first we need to know what the person means by abuse. In a person's thinking, it may be a thing that he has not seen his children for 3 days and that may be in context of his psychomotor poverty or retardation in context of schizophrenia or depression.

    Q16.

    A woman having individual psychodynamic psychotherapy with you comes in distressed saying she has had a dream about her father sexually abusing her. You:
    1. Acknowledge her distress and ask her to discuss her dream with you.
    2. Reassure her that she has not been abused.
    3. Tell her you can stop her father from abusing her by contacting the authorities.
    1. Acknowledge her distress and ask her to discuss her dream with you.




    Q17. Your colleague asks you what has been shown to predict a good response in psychodynamic psychotherapy. You respond:
    1. The patient’s perception of the working alliance with the therapist
    2. Young patients
    3. Intelligent patients
    1. The patient’s perception of the working alliance with the therapist





    Q 18. A man with moderate depression is unwilling to take antidepressants but will have psychotherapy. The factor which makes him suitable for brief psychodynamic psychotherapy is:

    1. He wishes to explore more about himself.
    2. There is a clearly defined psychodynamic focus.
    3. He is employed.
    4. He is less than 50 years old.

     
    1. He wishes to explore more about himself.
    Q19. A patient asks you “What is transference?” . The closest answer is: 1. Bringing past relationships into the therapy.


    Q 20. A patient asks you what “primary and secondary thoughts” are. You say:
    1. The primary thought is to guage the magnitude of a potential threat.
    2. The secondary thought is the negative automatic thought.
    3. The secondary thought gauges the magnitude of the threat.
    4. The secondary thought is the change in thought that comes through therapy.
    4. The secondary thought is the change in thought that comes through therapy.
     Q 21.  Bion – principle of container and containment
     a. Therapist advises the patient to contain the feelings without letting them out.
     b. Patient has to explore the feelings within and relate them to the surroundings.
     c. Therapist doesn’t let his feelings out.

     c. Therapist doesn’t let his feelings out and very importantly remains objective in his dealing with the patient.


    Q 22. The cognitive outcome you would hope to achieve in someone with hypochondriasis is:
    1. The complete absence of thoughts of illness.
    2. Better role functioning
    3. Less distress at thoughts of illness
    4. Better able to deal with frustration
    3. Less distress at thoughts of illness
    http://pn.psychiatryonline.org/cgi/content/full/39/10/38



    Q 23. In Learning disability and psychotherapy:
    1. Anxiety management therapy is usually done for people with profound LD.
    2. Gradual muscle relaxation is done in group psychodynamic therapy.
    3. Patients with LD who have been sexually abused are treated with brief psychodynamic therapy.
    4. Unmodified CBT is used in moderate LD.
    5. Guided mourning is not useful in mild LD.

    4. Unmodified CBT is used in moderate LD.
    Although more studies negate this, there are studies which provide substantial  evidence for use of CBT in people with LD.


    Child Psychiatry

    Q 24. Whilst working in child psychiatry you are dealing with a girl’s family. The girl has been abused by her father and he vehemently denies this abuse. The correct way for you to deal with the siblings of the abused child are:
    1. Have a family meeting with a suitable therapist without the abuser present.
    2. Have a family meeting with a suitable therapist and the abuser present.
    3. Have the siblings have a brief session with a psychotherapist.
    4. Residential treatment.
    5. Have the siblings see a child psychiatrist
    1. Have a family meeting with a suitable therapist without the abuser present. 

    Q 25. An otherwise fit and intelligent 15 year old boy has features of a moderate depressive illness. The correct treatment is:
    1. CBT
    2. SSRI
    3. CBT + SSRI
    4. TCA
    1. CBT 

    26. In children, after pervasive development disorders,
    the next illness with a chronic poor prognosis is:
    1. Bipolar affective disorder
    2. Schizophrenic psychosis
    3. Social phobia
    4. Drug induced psychosis
    1. Bipolar affective disorder 

    http://www.ncbi.nlm.nih.gov/pubmed/17452221




    Q 27. What treatment would you give an 8 year old with hyperkinetic disorder as first line treatment (with no other comorbidities):
    1. Methylphenidate
    2. Atamoxetine
    3. Dexamphetamine
    4.Clonidine
    5.Imiprimine
    1. Methylphenidate 

    Q 28. In a child with Tourettes and Hyperkinetic disorder the best treatment is:
    1. A central alpha agonist
    2. A NARI
    3. A TCA
    4. Pemoline
    5. None of the above


    Ans: 1 A central alpha agonist  (clonidine)


    Q 29.What treatment is not indicated for a child with hyperkinetic disorder without any other comorbidity.

    1.Pemoline
    2. Atomoxetine
    3.Imipramine
    4. Clonidine
    5. Both 3 and 4

    Ans:
    1. Pemoline (Due to hepatoxicity)
    SPMM course.
    Q 30. In a child with PANDAS you would expect to find raised:
    1. anti-DNAases
    2. ceruloplasmin
    3. anti-nuclear antibodies
    4. Antimicrosomal antibodies
    5. ANCA (Anti-Neutrophilic Cytoplasmic Antibodies)
    1. anti-DNAase antibodies 

     if "ASO antibodies" is within options, go for it as a first option.

    From following link
    http://intramural.nimh.nih.gov/pdn/web.htm

    Q. What exactly is an anti-streptococcal antibody titer?

    A. The anti-streptococcal antibody titer determines whether there is immunologic evidence of a previous strep. infection. Two different strep. tests are commercially available: the antistrepolysin O (ASO) titer, which rises 3-6 weeks after a strep. infection, and the antistreptococcal DNAase B (AntiDNAse-B) titer, which rises 6-8 weeks after a strep. infection.
    Q 31. A 12 year old is brought to see you due to shoplifting. He is most likely to have:
    1. Conduct disorder
    2. Depression
    3. Oppositional Defiant Disorder
    4. Hyperkinetic Disorder

    Ans 1 Conduct Disorder






    Statistics 

    Q 32. In the following figure, which number represents the point with the best mixture of sensitivity and some specificity?
    Here is what the graph of the ROC curve would look like.
    Options:
    1)  0.4
    2) 0.67
    3)  0.91
    4)  0.97
    5)  1.0
    Answer:
    3)  0.91
    Here is information about Area Under the Curve. This area (0.91) is quite good; it is close to the ideal value of 1.0 and much larger than worst case value of 0.5
    __________________________________________

    Q 33.  Which figure is this?
    1. A Receiver Operator Characteristics curve
    2.A Galbraith plot
    3.A Funnel plot
    4.A Scattergram
    5. A stem-leaf plot
    Answer:
    1. A Receiver Operator Characteristics curve
    ________________________________________________

    Q 34. There was a normogram and we are given the pretest probability and the likehood ratio of a negative result. We are also given the ?specificity of the test and asked to calculate the post-test probability from the normogram.



    Q 35. Which of the following reduces bias in a test:
    1. Block randomisation
    2. Intention to treat analysis
    3. Stratification
    4. Minimisation
    5. All of above
    5. All of above
     Ref. Ajetunmobi;  Gosall and Gosall

    Q 36.  What does not reduce confounding:
    1. Mantzel-haentzel procedure
    2. Regression
    3. Blinding
    4. Restriction
    5. Matching
    3. Blinding
    Ref. Ajetunmobi

    Q 37. What increases the power of a study:

    1. Comparing active treatment vs placebo
    2. Comparing active treatment vs active treatment
    3. Smaller numbers of participants
    4. Increased heterogeneity
    5. All of the above 
    Answer:
    1. Comparing active treatment vs placebo

    Liaison and Eating Disorders

    Q 38.  You have a female patient with bilateral herpetic encephalopathy now presentling with increased sexual drive and carbohydrate craving. She has:

    1. Kluver-Bucy Syndrome
    http://www.pubmedcentral.nih.gov/articlerender.fcgi?artid=1032530

    Q 39. A 40 year old farmer comes to see you with general malaise, low mood, and a circumscribed 4cm red lesion on his chest. You diagnose:
    1. Chronic fatigue syndrome
    2. Lyme disease
    3. Huntington’s disease
    Answer: 2. Lyme Disease
    http://www.nature.com/nrmicro/journal/v3/n2/box/nrmicro1086_BX1.html

    Q 40. What treatment would you use for pathological crying?
    1. Citalopram
    2. Fluoxetine
    3. Paroxetine
    4. Venlafaxine
    5. Sertraline.
    1.    Citalopram 
    http://www.ncbi.nlm.nih.gov/pubmed/8104273

    Q 41.  What treatment has the most evidence for depression following a myocardial infarction:

    1. Citalopram
    2. Fluoxetine
    3. Paroxetine
    4. Venlafaxine
    5. Sertraline
    5. Sertraline
    http://www.ncbi.nlm.nih.gov/pubmed/12169073        

    Q 42.  What treatment would you use for a patient with severe symptoms of pre-menstrual syndrome:

    1. Citalopram
    2. Fluoxetine
    3. Paroxetine
    4. Venlafaxine
    5. Sertraline
    2. Fluoxetine

    http://www.emedicine.com/ped/topic1890.htm  





    Q 43. What treatment has been shown to be of benefit in women with premenstrual syndrome:


    1. Light therapy
    2. Evening primrose oil
    3. SSRIs
    4. Oligoacids
    3.    SSRIs

    Ref. Puri Revision Notes

    Q 44. A lady with multiple sclerosis presents with depression. The drug most likely to be the cause is:
    1. Baclofen
    2. Beta interferon
    2. Beta interferon

    Q 45.  Which drug given prophylactically is shown to decrease delirium:
    1. Quetiapine
    2. Amisulpiride
    3. Haloperidol
    4. Olanzapine
    5. Risperidone


    Q 46.  Which of the following statements with regards to offending in learning disabilities is INCORRECT:
    1. It is overrepresented by borderline and mild LD.
    2 Fire setting is the most common offense.
    3. People with moderate LD are more likely to commit violent offences.
    1. It is overrepresented by borderline and mild LD.

    Q 47. The prevalence of psychotic disorders in prisons compared to the general population is:
    1. 5 times
    2. 10 times
    3. 20 times

    4. 40 times                                                                                                                                 
    5. 50 times
    3. 20 times

    Q 48. What percentage of people charged with sexual offences have no previous history of sexual offences?
    1. 10%
    2. 20%
    3. 40%
    4. 50%
    5. 80%

    From a sample of almost 21,000 prisoners
    discharged in 1995, 8,626 offenders had no previous or current history of sexual or violent offences (41%). http://www.nationalarchives.gov.uk/ERORecords/HO/415/1/rds/pdfs/r104.pdf 

    About one third of the two control groups (32% and 35%) and two thirds of the treated group (63%) had previous sexual convictions. http://66.102.9.104/search?q=cache:ROa6OT83wI4J:www.csc-scc.gc.ca/text/pblct/forum/e052/e052c-eng.shtml+People+sexual+offences+percent+No+previous+history&hl=en&ct=clnk&cd=15&gl=pk 

    Q 49. What is the best predictor of future sexual offences?
    1. Previous non-sexual offences
    2. Lack of victim empathy
    3. Mental illness
    4. History of illicit drug use
    1. Previous non-sexual offences            http://bjp.rcpsych.org/cgi/content/full/190/3/223   


    Q 50. What percentage of people charged with sexual offences have no previous history of sexual offences?
    1. 10%
    2. 20%
    3. 40%
    4. 50%
    5. 80%

    From a sample of almost 21,000 prisoners
    discharged in 1995, 8,626 offenders had no previous or current history of sexual or violent offences (41%). http://www.nationalarchives.gov.uk/ERORecords/HO/415/1/rds/pdfs/r104.pdf 

    About one third of the two control groups (32% and 35%) and two thirds of the treated group (63%) had previous sexual convictions. http://66.102.9.104/search?q=cache:ROa6OT83wI4J:www.csc-scc.gc.ca/text/pblct/forum/e052/e052c-eng.shtml+People+sexual+offences+percent+No+previous+history&hl=en&ct=clnk&cd=15&gl=pk



    Q 51. What is the best predictor of future sexual offences?
    1. Previous non-sexual offences
    2. Lack of victim empathy
    3. Mental illness
    4. History of illicit drug use
    1. Previous non-sexual offences            http://bjp.rcpsych.org/cgi/content/full/190/3/223   
    Blog EntryAug 20, '08 11:12 AM
    for everyone
    Q 52. People with emotionally unstable borderline personality disorder are stuck in:
    1. Winnicott’s “false self”
    2. Klein’s depressed position
    3. Klein’s paranoid-schizoid position
    3. Klein’s paranoid-schizoid position
    Winnicott’s false self relates to schizoid personality disorder and not borderline personality disorder
    Q 53.  Something that is diagnostic of borderline personality disorder is:
    1. Attempts to avoid real or imagined abandonment.                                                                       
    2. Depression
    3. Suicide
    4. Self-harm
    1. Attempts to avoid real or imagined abandonment.                                                         

    Q 54.  A young man is impulsive and has angry outbursts. He has been arrested several times for hitting his partner. His diagnosis is:
    1. Dissocial PD
    2. Emotionally Unstable PD
    3. Paranoid PD
    4. Anankastic PD
    1. Dissocial Personality Disorder

    Q 55. A man comes to your out-patient clinic. He gives you a history in childhood of conduct disorder. He recognises that he has a problem with anger and often gets into fights with his partner. He wishes to prevent this from happening. You interview him and find no evidence of mental illness or active symptoms. You decide:
    1. To start no treatment but review him in three months.
    2.Start an antipsychotic and review in 3 months.
    3. Start an antidepressant and review in 3 months.
    4. To discharge him.
    1. To start no treatment but review him in three months.


    Mood and Anxiety disorders
    Q 56.  A man comes to your clinic nine months after the death of his mother. You find features of a moderate depressive illness. He occasionally hears her voice calling him. You decide to:
    1. Do nothing and reassure him it will all go away.
    2. Start an antidepressant and follow him up.
    3. Start an antipsychotic and follow him up.
    2. Start an antidepressant and follow him up.
    http://focus.psychiatryonline.org/cgi/content/abstract/2/2/260

    Q 57. Social phobias are most associated with:
    1. Generalising to other phobias
    2. Avoidance to reduce fear
    3. Obsessions
    4. Compulsions to reduce anxiety
    2. Avoidance to reduce fear                                                                                                  
    Q 58. In generalised anxiety disorder we see:
    1. Briefly punctuated by episodes of depression
    2. Rituals
    3. Obsessions
    4. Suicide
    5. Higher dizygotic twin concordance rather than monozygotic.

    1. Briefly punctuated by episodes of depression
    http://www.cnn.com/HEALTH/library/DS/00502.html  

    Q 59. A woman comes into your outpatient clinic. She is obsessed with dust and has to wash her hands up to twenty times if she touches anything. The treatment you would recommend would be:
    1. Response and Exposure prevention (Not exposure and response prevention!)
    2. CBT
    3. CAT
    4. Psychodynamic therapy
    5. Interpersonal therapy
    2. CBT
    CBT includes Exposure and Response Prevention

    Q 60. What would you do for a patient with OCD and no improvement after two months on 50mg of sertraline:
    1. Increase the sertraline
    2. Change to a different SSRI
    3. Add an antipsychotic
    4. Switch to venlafaxine
    5. Add sodium valproate
    1.    Increase the sertraline


    Addiction Psychiatry

    Q 61. You are the substance misuse consultant and are asked to draw up a service for users. The first thing to do will be:

    1. Address lifestyle imbalances

    1. Address lifestyle imbalances
    Q 62. Acamprosate:
    1. Can be used with controlled drinking.
    2. Is associated with abuse potential.
    3. Acts in a dose-dependent fashion.
    4. Reduces craving for alcohol.
    4. Reduces craving for alcohol.
    No. 1 is also true, but 4 is the best fit.



    Q 63.  You are called to A&E to see a man withdrawing from amphetamines. He is least likely to have:
    1. Insomnia
    2. Seizures
    3. Hypersomnia
    4. Agitation
    5. Decreased appetite

    2. Seizures

    Q 64. You are called to A&E to see someone who has taken PCP. The following are true:
    1. Gait disturbances are late
    2. Visual hallucinations are common
    3. None of the above.

    1. Gait disturbances are late 
      

    Q 65. A 19 year old tells you she takes MDMA. The least likely psychological consequence of this drug is:
    1. Anxiety
    2. Disinhibition
    3. Increased desire to do mental tasks
    4. Perceptual disturbances
    5. Increased friendliness

    1. Anxiety 

    Q 66. A man who drinks two bottles of vodka a day tries to stop by himself. He is brought into hospital with ataxia, and confusion. The chances that he will develop Korsakov’s are:
    1. 10%
    2. 20%
    3. 50%
    4. 80%
    5. 100%
    3.    80% (SOTP)

    Perinatal psychiatry

    Q 67. A patient of yours is breastfeeding. She has Bipolar Affective Disorder and came off her medication for her pregnancy. She feels she is about to relapse. What medication is safe in breast feeding:
    1. Choral hydrate
    2. Lamotrigine
    3. Sodium valproate
    4. Lorazepam
    5. Carbemazepine

    3.    Sodium valproate
    Maudsley Prescribing guidelines


    Q 68.  A female patient has had several depressive episodes in the past and one episode of hypomania. You decide the best medication to prevent relapse for her is:
    1. Lithium
    2. carbemazepine
    3. Lamotrigine
    4. Fluoxetine
    5. Sodium valproate
    Lithium
    Maudsley Prescribing guidelines

    Q 69. A woman comes to see you in out-patients. She has a 6month old son and for the last three months she has recurring thoughts of harming him. She does not wish to harm him and these thoughts make her tearful and anxious. The birth was uneventful but she perceives it to have been traumatic. You diagnose:
    1.PTSD
    2. Postnatal depression
    3.OCD
    2. Postnatal depression
    If tearfulness and so called history of traumatic birth is not there, the answer can also be OCD. But with the history given, it is postnatal depression.

    Psychiatry for Older Adults

    Q 70. How do you differentiate that a dementia is subcortical not cortical:
    1. Calculation is preserved
    2. Apathy

    1. Calculation is preserved
    http://home.epix.net/~tcannon1/depressionvsdementia.htm  
      
    In subcortical dementia cortical functions such as calculation, are preserved till late and go away later.

    Q 71.  In an 80 year old patient compared to a 40 year old patient you would expect the half-life of temazepam to be:
    1. Reduced by 50%
    2. The same
    3. Increased by 50%
    4. Increased by 100%
    5.Increased by 200%

    5.Increased by 200%
    Starting dose in elderly is 7.5 mg as compared to 15 mg in a 40 year adult.


    Rating Scales

    Q 72. You have started a patient with postnatal depression on an antidepressant and wish to monitor changes in her symptoms. Which of the following observer rating scales would you use:
    1. Zung
    2. BDI
    3. MADRS
    4. Morgan-Russel
    5. Edinburgh Depression Scale
    3. MADRS
    Zung, BDI and EDRS are self rated, Morgan Russell is for anorexia nervosa.
    EDRS is a screening tool.


    Q 73.  A young man in your clinic complains of EPSEs. The scale you would use to measure this is:
    1. Simpson-Angus
    2. Morgan-Russell
    3. Young Mania Scale
    4. AIMS (Abnormal Involuntary Movements Scale)
    5. PANSS (Positive & Negative Symptom Scale)

    1. Simpson-Angus

    Q 74. What rating scale would you recommend for a health visitor to use to screen for depression in women after delivery:

    1. Zung
    2. BDI
    3. MADRS
    4. Morgan-Russel
    5. Edinburgh Depression Scale

    Edinburgh Depression Scale


    Pharmacology
    Q 75. A male patient on clozapine has gained 20kg. Your next step is to:
    1. Switch to Quetiapine.
    2. Switch to Risperidone
    3. Add sodium valproate
      4. Dietary Counselling
    5. Switch to amisulpiride

    44. Dietary counselling
    Maudsley Prescribing Guidelines
    Other options can be to add amantadine or to switch to ziprasidone, aripiprazole or haloperidol.


    Q 76. What drug causes symptoms of inappropriate ADH secretion:
    1. Quetiapine
    2. Amisulpiride
    3. Haloperidol
    4. Olanzapine
    5. All of above
    5. All of above
    MAUDSLEY

    Q 77.  A man with Parkinson’s Disease develops psychotic symptoms. He has never been on antipsychotics before. The best treatment for him is:
    1. Quetiapine
    2. Amisulpiride
    3. Haloperidol
    4. Olanzapine
    5. Risperidone

    1. Quetiapine
    Maudsley p.426

    Q 78.  A female patient has been trialled on Olanzapine and Quetiapine with poor effect. Your next step is to try:
    1. Clozapine
    2. Amisulpiride
    3. Haloperidol
    4. Lithium
    5. Aripiprazole

    1.               Clozapine
    Set 2

    Q 79.  Which of the following is commonly associated with microcephaly? 
    1. Foetal Alcohol Syndrome
    2. Hydrocephalus
    3. Soto’s Syndrome
    4. Down’s Syndrome
    5. Fragile X Syndome
    1.               Foetal Alcohol Syndrome
    Q 80.  Which of the following is true of offending in the learning disability population?

    1. Fire setting is the most commonly committed offence
    2. Most offences are committed by those with borderline and mild learning disabilities
    3. Moderate LD is strongly associated with homicide
    4. Conviction for arson leads to a fixed prison sentence
    5. All of above
    . Most offences are committed by those with borderline and mild learning disabilities 


    Q 81.  Which of the following increases the risk of Alzeihmer’s Disease in Down’s Syndrome?

    1. Positive family history
    2. Mild LD
    3. Moderate LD
    4. Smoking
    5. Aluminium exposure
    1. Positive family history

    Q 82.  Which of the following is true of psychotropic medications in the learning disability population?
    1. Ethosuxamide is a first line antiepileptic
    2. Lamotrigine has mood stabilising effects
    3. Valproate leads to weight loss
    4. Proprananol is an effective antimanic agent
    5. Naltrexone is an effective antidepressant

    2. Lamotrigine has mood stabilising effects


    Q 83. In Learning disability and psychotherapy:
    1. Anxiety management therapy is usually done for people with profound LD.
    2. Gradual muscle relaxation is done in group psychodynamic therapy.
    3. Patients with LD who have been sexually abused are treated with brief psychodynamic therapy.
    4. Unmodified CBT is used in moderate LD.
    5. Guided mourning is not useful in mild LD.
    Although more studies negate this, there are studies which provide substantial  evidence for use of CBT in people with LD.
    4. Unmodified CBT is used in moderate LD.


    Q 84.  Which of the following is most associated with reoffending in paedpophiles?

    1. lack of victim empathy
    2. psychiatric illness
    3. comorbid substance misuse
    4. previous violent offending
    5. depressed mood

    4. lack of victim empathy 
    The strongest static predictors were
    age/gender/race and criminal history, and the strongest dynamic predictors were
    "criminogenic need" factors, which they defined as antisocial personality, antisocial
    companions, interpersonal conflict, antisocial attitudes, and substance abuse (Gendreau et al.,
    1996: 583-584).

    Q 85.  Which of the following is used as a screening tool on psychiatry?

    1. Simpson Angus Scale
    2. MADRAS
    3. SANS
    4. Edinburgh Post Natal Depression Scale
    5. BDI

    4. Edinburgh Post Natal Depression Scale

    Q 86.  Which of the following is associated with tall stature?

    1. Prader Willi Syndrome
    2. Williams Syndrome
    3. XXY
    4. Foetal Alcohol Syndrome
    5. Cornelia De Lange Syndrome
    3. XXY
    Both XXY and XYY are associated with tall stature. Ref. College Seminars in LD.

    Q 87.  Which of the following is a recognised symptom of severe depression

    1. parasomnia
    2. somnambulism
    3. hypersomnia
    4. narcolepsy
    5. night terror
    3. hypersomnia


    Q 88. One of the following is not a feature of chronic fatigue syndrome
    a. Disturbed sleep pattern
    b. Feeling tired after rest, relaxation and enjoyment
    c. Duration more than 6 months
    d. Exercise makes person more tired
    e. Muscle aches and pains.
    c. Duration more than 6 months
    Nowhere mentioned in ICD-10 about duration (F48.0)

    Q 89.  Bion – principle of container and containment
    a. Therapist advises the patient to contain the feelings without letting them out.
    b. Patient has to explore the feelings within and relate them to the surroundings.
    c. Therapist doesn’t let his feelings out.
    c. Therapist identifes and interprets transference and safely reprojects onto the group / client. 

    Blog EntryAug 19, '08 11:18 AM
    for everyone
     Set 3


    Q 91 • Stem: What is true about the initial appraisal of an event
    Options:
    Automatic thoughts being part of primary appraisal
    Perception of the magnitude of the threat being part of primary appraisal
    Negative automatic thoughts being part of secondary appraisal
    Perception of the magnitude of the threat being part of secondary appraisal

    Automatic thoughts being part of primary appraisal


    Q 92  • What is transference ?
    Options:
    Patient’s response to the therapist based on previous relationships
    Therapist’s response to the patient
    Empathy in relationship
    Patient’s response to the therapist based on previous relationships
    Q 93 • What predicts a good response to therapy?
    Options:
    Good perception of treatment alliance
    Patient can understand things in psychological terms
    Previous therapy

    Good perception of treatment alliance

    Q 94.  • What is a core feature of emotionally unstable personality disorder
    Options:
    Attempts to avoid abandonment
    Impulsive acts
    Poor self esteem

    Attempts to avoid abandonment

    Q 95. • What defence mechanism is best explained by a woman who says she is happy but the world and everyone in it seems depressed
    Options
    Overgeneralisation
    Externalisation
    Projective identification

    Overgeneralisation

    Q 96.  • What theorist best explains someone with emotionally unstable personality disorder?
    Options
    Winnicotts false self
    Klein’s depressive position
    Klein’s paranoid schizoid position
    Klein’s paranoid schizoid position

    Q 97.  • Stem: At the second session of psychodynamic psychotherapy, a distressed young woman says she dreamed she was sexually abused by her father. What do you do?
    Options:
    Acknowledge her distress and ask her to talk about the dream
    Acknowledge her distress and ask if she was abused by her father
    Leave it to the next session
    Explore the feelings it arouses in you

    Acknowledge her distress and ask her to talk about the dream

    Q 98.  • Stem: During therapy a male patient tells you that he has been sexually abusing children. He immediately starts talking about an entirely different matter. What do you do?
    Options:
    Explain that you have to contact the relevant authorities and do so
    Ask him to say more about what he means by ‘abuse’
    Change the subject
    Stop the therapy at the correctly appointed time

    Explain that you have to contact the relevant authorities and do so

    Q 99.  • What did Bion mean by ‘containment’ in terms of therapy?
    Options:
    The therapist identifies the projected transference and projects it back safely
    The therapy is contained within a timed framework
    The therapist is able to deal with emotions as part of therapy
    The therapist identifies the projected transference and projects it back safely

    Q 100.  • Stem: Tall stature is associated with
    Options:
    Cornelia de Lange syndrome
    Turner’s syndrome
    XYY
    Prader-Willi syndrome
    Lesch Nyhan
    XYY
    (Both XXY and XYY are associated with tall stature)


    Q 101.• Stem: what is the incidence of birth defects in a woman taking lithium
    Options:
    1/10
    1/100
    1/1000
    1/1000

    Maudsley Prescribing Guidelines

    Q 102 • What is the incidence of birth defects in a woman taking valproate?
    Options:
    1/10
    1/50
    1/100
    1/1000
    1/100
    - Risk associated with Epilim
    In animals: teratogenic effects have been demonstrated in the mouse, rat and rabbit.
    There is animal experimental evidence that high plasma peak levels and the size of an individual dose are associated with neural tube defects.
    In humans: Available data suggest an increased incidence of minor or major malformations including neural tube defects, craniofacial defects, malformation of the limbs, cardiovascular malformations and multiple anomalies involving various body systems in offspring born to mothers with epilepsy treated with valproate when compared to the incidence for certain other antiepileptic drugs. Epilim use is associated with neural tube defects such as myelomeningocele and spina bifida. The frequency of this effect is estimated to be 1 to 2%. Data suggest that antiepileptic polytherapy including Epilim induces a higher teratogenic risk than monotherapy with valproate only. Data have suggested an association between in-utero Epilim exposure and the risk of developmental delay particularly of verbal IQ, genetic, social, environmental factors and poor maternal seizure control during pregnancy.  
    http://64.233.183.104/search?q=cache:oZKsAUT_2P8J:www.mentalhealth.com/drug/p30-d02.html+Valproate+birth+defects&hl=en&ct=clnk&cd=6&gl=pk

    Q 103  • What is the relative risk of psychosis in prisons?
    Options:
    5
    10
    20
    100
    10 times more common than in general population

    Q 104  • A man wants to take a herbal antidepressant. What herb would you recommend?
    Options:
    Hypericum perforatum
    Q 105 • What characterises GAD
    Options:

    Avoidance to reduce anxiety
    Depressive episodes



    Q 106  • What characterises phobias?
    Options:
    Avoidance to reduce anxiety
    Generalisation of phobia


    Q 107 • What rating scale would you use in a woman who had given birth recently and appeared depressed in order to assess her response to antidepressants?
    Options:
    MADRS
    Edinburgh postnatal
    HAMD
    BPRS
    MADRS

    Q 108 • What is a screening tool used in psychiatry?
    Options:
    Edinburgh postnatal

    Q 109 • Stem: Which is the first line drug in an 8 year old with uncomplicated ADHD?
    Options:
    Clonidine
    Atomoxetine
    Imipramine
    Dexamphetamine
    Methylphenidate
    Methylphenidate
    Q 110 • Stem: Which class of drug would you use in a boy with ADHD and Tourette’s?
    A centrally acting alpha agonist
    Beta blocker
    Noradrenaline reuptake inhibitor
    A centrally acting alpha agonist
    http://books.google.com.pk/books?id=-j-ES1O_J4gC&pg=PA141&lpg=PA141&dq=adhd+tourette+treatment+of+choice&source=web&ots=a-GdpJw6D2&sig=BtpCZimHvFrFl6AxIELZBL9Hu10&hl=en 

    Q 111 • Stem: Concerning learning disability and offending
    Options:
    Most offences are committed by those with mild/mod LD
    Fire setting is the most common offence
    Those with moderate LD are most likely to be convicted of murder
    Sexual offending is related to hypersexuality

    Most offences are committed by those with mild/mod LD

    Q  112 • Stem: Select one correct statement regarding psychotherapy in LD
    Options:
    Unmodified CBT can be used in mild to moderate  LD
    Guiding mourning has no place in mild LD
    Progressive relaxation is part of psychodynamic group therapy
    Patients with severe LD who have been traumatized by abuse should have psychodynamic therapy
    Behaviour therapy should be used in groups in severe LD

    Unmodified CBT can be used in mild to moderate  LD


    Q 113  • Stem: Which of the following is true concerning psychotropic medication in those with LD?
    Options:
    Lamotrigine has a mood stabilizing effect
    Propanolol is an effective antimanic agent
    Naltrexone is an effective antidepressant
    Clonazepam causes agitation
    Lamotrigine has a mood stabilizing effect


    Q 114 • Stem: How would you treat an intelligent 15 year old boy with moderate depression but no suicidal thoughts?
    Options:
    CBT
    SSRI and CBT
    TCA alone
    TCA and CBT
    SSRI alone
    CBT

    Q 115 • Apart from autistic spectrum disorders which of the following disorders has the worst prognosis if onset is in adolescence?
    Schizophrenia
    BPAD
    Schizoaffective disorder
    Psychotic depression

    Schizophrenia

    Q 116 • Stem: In children with PANDAs which symptoms are least common?
    Options:
    Obsessions
    Depression
    Auditory hallucinations
    Anxiety
    Auditory hallucinations

    http://www.adhd.com.au/PANDAS.htm     http://www.tourettesyndrome.net/pandas_overview.htm http://www.webpediatrics.com/pandas.html


    Q 117 • Stem: In PANDAs which of the following blood tests is most likely to be positive?
    Options:
    Anti DNAse antibodies
    Anti nuclear antibodies
    IgA
    Anti DNAse antibodies

    Q 118• Stem: Truancy is most associated with
    Options:
    Conduct disorder
    Oppositional defiant disorder
    ADD
    Depression
    Encopresis
    Conduct disorder
    Q 119 • Stem: Empathy skills are most likely to be delayed in
    Options:
    A deaf child of deaf parents
    A deaf child of hearing parents
    A hearing child of deaf parents
    A hearing child with one deaf parent
    A hearing child adopted at birth


    A deaf child of hearing parents


    http://64.233.183.104/search?q=cache:bJ3CdSLVKDoJ:www.aare.edu.au/96pap/nichs96342.txt+empathy+deaf+child+hearing+parents&hl=en&ct=clnk&cd=13&gl=uk

    Q 120 • Which of the following increases the risk of Alzheimer’s disease in Down’s syndrome?
    Severe LD
    Mild LD
    Family history of Alzheimer’s
    Other brain pathology
    Family history of Alzheimer’s

    Q 121 • Stem: Microcephaly is a characteristic feature of
    Options:
    Down’s syndrome
    Foetal alcohol syndrome
    Fragile X syndrome
    Hydrocephalus
    Soto’s syndrome
    Foetal alcohol syndrome

    Q 122 .  Stem: Diagnostic features of neurasthenia include the following except:
    Easy fatiguability after minimal effort
    Poor (unrefreshing) sleep
    Muscular aches and pains
    Duration >6 months

    Duration >6 months


    Q 123 • Stem: Which of the following has no evidence of treatment in PTSD?
    Options:
    EMDR
    Hypnotherapy
    Stress management
    Trauma focused CBT
    Group psychotherapy

    Hypnotherapy
    Q 124 • Stem: Huntington’s disease shows which pattern of inheritance
    Options:
    Autosomal dominant with high penetrance
    Autosomal dominant with poor penetrence
    Autosomal recessive with low penetrance
    X-linked
    Autosomal dominant with high penetrance
    http://www.emedicine.com/emerg/topic254.htm


    Q 125• Which antidepressant has the best evidence base for its use in post MI depression?
    Fluoxetine
    Reboxetine
    Mirtazapine
    Sertraline
    Imipramine
    Sertraline

    Q 126 • Stem: Which antidepressant has the best evidence base for its use in post stroke depression?
    Options:
    Fluoxetine
    Citalopram
    Reboxetine
    Mirtazapine
    Sertraline
    Fluoxetine 
    Other good agent is nortriptyline which has the best evidence for post-stroke depression. Use sertraline if history of recent MI. Use citalopram if patient is on warfarin. 
    References:
    1) Maudsley Prescribing guidelines


    Q 127 • Stem: Which of the following treatments for MS is most likely to lead to depression?
    Options:
    Amantadine
    Baclofen
    Beta-interferon
    Steroids
    Beta-interferon


    Q 128 • Stem: A woman who has had Herpes encephalitis develops severe carbohydrate craving and weight gain. Diagnosis?
    Options:
    Cushing’s
    Diabetes mellitus
    Hypothyroidism
    Kluver-Bucy syndrome
    Prader Willi syndrome
    Kluver-Bucy syndrome

    Q 129 • Stem: Which of the following is most common in delirium?
    Options:
    Hallucinations
    Delusions
    Disturbed sleep wake cycle
    Labile mood
    Increased motor activity
    Disturbed sleep wake cycle


    Q 130 • Stem: Which of the following are most likely to be seen in anorexia
    Option:
    High oestrogen
    Low cortisol
    High white cell count
    Low triiodothyronine
    Hyperkalaemia
    Low triiodothyronine
    http://books.google.com.pk/books?id=LRALl8IA4hoC&pg=PA22&lpg=PA22&dq=anorexia+nervosa+thyroid&source=web&ots=7FwjGZNaOp&sig=Cy91F7rTqyNR3rJoH9oa-DP3pyE&hl=en 

    Q 131 • Stem: The babies of anorexic mothers are most likely to be
    Options:
    Are large for dates
    Have lower APGAR scores
    Are born post-term
    Have a larger head circumference
    Have fetal abnormalities

    Have lower APGAR scores

    http://ajp.psychiatryonline.org/cgi/content/full/158/9/1461 


    Q 132 • Stem: Which of the following is least commonly associated with bulimia nervosa?
    Options:
    Oesophageal tears
    Dental decay
    Peptic ulcer
    Parotid gland enlargement
    Seizures
    Seizures
    Q 133 • Stem: Normal bereavement is most likely to include
    Options:
    Delusional beliefs that the deceased is still alive
    Significant weight loss
    Suicidal ideation
    Worthlessness
    Transient anger to the deceased

    Transient anger to the deceased
    http://www.psychnet-uk.com/dsm_iv/bereavement_symptoms.htm  


    Q 134 • Stem: Which of the following has the best evidence for its use in pre-menstrual syndrome?
    Options:
    Phenelzine
    Reboxetine
    Amitryptalline
    Fluoxetine
    Venlafaxine
    Fluoxetine


    Q 135 • Stem: Which of the following has the best evidence of use in pre-menstrual dysphoria?
    Options:
    Progesterone
    SSRIs
    Vitamin B6
    Bright light therapy
    Oil of evening primrose
    SSRIs

    Q 136 • Stem: Which of the following is contraindicated in the elderly with psychosis and cognitive impairment?
    Options:
    Haloperidol
    Quetiapine
    Olanzapine
    Sulpiride
    Promazine
    Olanzapine
    Blog EntryAug 18, '08 10:32 PM
    for everyone
    EMI 1  
    Couvade Syndrome
    Capgras  Syndrome
    Cotard’s Syndrome
    Fregoli Syndrome
    De Clerambault  Syndrome
    Othello Syndrome
    1. An elderly woman has insomnia, low mood and weight loss. She tells you there is no point in her eating anything as her insides are rotting – Choose ONE
    Cotard’s syndrome
    2. A young man whose wife is pregnant complains of morning sickness and an inflated abdomen – choose ONE
    Couvade Syndrome
    3. A woman with a son says that lately he tells her that her male friend has been replaced by an imposter. She keeps telling him this is not the case but he does not believe her. She has had to ask her son to leave her home because of this.
    Capgras Syndrome
    EMI 2

    OPTIONS
    Increased physical dependence Being in an institution
    Male patient
    Male carer
    Dementia
    Being married to carer
    Previous poor relationship with carer

    QUESTIONS
    1) 3 risk factors for abuse

    Dementia
    Previous poor relationship with carer
    Being married to carer

    2) 3 risk factors for depression

    Male patient
    Increased physical dependence
    Dementia

    3) 2 risk factors for carer exhaustion

    Previous poor relationship with carer
    Male patient
    EMI 3
    1.Neuroleptic malignant syndrome
    2. Cardiomyopathy
    3. Diabetes Insipidis
    4. Diabetes Mellitus
    5. Cushing’s Syndrome
    6. DKA
    7. Delirium Tremens
    8. Lithium toxicity
    1. A young man on clozapine complains of breathlessness and has a persistent tachycardia. Choose ONE.
    2. Cardiomyopathy
    2. A woman on lithium for many years who recently complains of feeling weak. Her serum sodium is raised and she looks dehydrated.
    3. Diabetes Insipidis
    3. A young man started on an antipsychotic 2 weeks ago now complains of stiff muscles, fever , labile BP, and confusion.
    1.Neuroleptic malignant syndrome
    EMI 4
    Alcoholic hallucinosis
    Fahr’s syndrome
    Alcoholic delirium
    Alcohol withdrawal
    Meta-chromatic leucodystorphy
    Neuroacanthosis
    Systemic Lupus Erythematosis
    Complex partial seizures
    Post Herpetic encephalopathy
    1. A 44 year old homeless man is brought into A&E. he is agitated, psychotic and depressed. The ECG shows decreased alpha activity and his gait is disturbed. MRI shows hypo-intensity of the striatum. Choose ONE.
    Fahr’s syndrome
    2. A 25 year old man recently returned from America shows aggressive behaviour. He recently developed seizures and has periods of decreased conciousness. His full blood count and liver functions are normal – Choose ONE.
    Post Herpetic encephalopathy
    3. A 30 year old woman has one sided facial nerve palsy and circumscribed lesions on both legs. Choose ONE.
    Post Herpetic encephalopathy
    4. A 44 year old homeless man is brought into A&E. he is agitated, psychotic and depressed. The ECG shows decreased alpha activity and his gait is disturbed. MRI shows hypo-intensity of the corpus callosum. Choose ONE.
    Marchiafava-Bignami Disease
    EMI 5
    Universality
    Pairing
    Dependence
    Fight-flight
    Cohesiveness
    Free floating discussion
    Interpreting transference
    Counter-dependence
    1. Two factors that are curative in groups.
    Universality
    Cohesiveness
    2. Two factors that hinder working in groups (Basic Assumptions)
    Pairing
    Dependence
    Fight-flight
    3. Two factors that are found in psychodynamic groups.
    Free floating discussion
    Interpreting transference
    Box 2. Yalom's curative factors (Yalom, 1970)
    1. Interpersonal learning
    2. Catharsis
    3. Group cohesiveness
    4. Self-understanding
    5. Development of socialising techniques
    6. Existential factors
    7. Universality
    8. Instillation of hope
    9. Altruism
    10. Corrective family re-enactment
    11. Guidance
    12. Identification/imitative behaviour
    EMI 6
    Options
    A.Family therapy
    B. Methylphenidate
    C. Parenting skills training
    D.Multisystematic Therapy
    1. A child whose mother says he is disobedient and defiant.
    C. Parenting skills training
    Ref: Child Psychiatry A Developmental Approach 3rd edition
    2. A 17 year old whose parents have taken out a restraining order against because of his violence to them.
    C. Risperidone
    References:
    Child Psychiatry A Developmental Approach 3rd edition
    3. A child who has been diagnosed with hyperkinetic disorder by the psychologist.
    B. Methylphenidate
    EMI  7
    No psychiatric problem
    Alcohol Withdrawal
    Delirium Tremens
    Alcohol dependence
    Dementia
    Korsakof’s psychosis
    Wernicke’s encephalopathy
    Oriental Flush
    1. A young Japanese student whose face flushes and has nausea after two pints of beer.
    Oriental Flush

    Inside story: the body

    File Format: PDF/Adobe Acrobat - View as HTML
    Oriental flush. Half of all people of Japanese or Chinese. cultures are made quite ill even by small. amounts of alcohol. ...
    www.sycd.co.uk/who_am_i/pdf/inside_body/body_alcohol_pupils.pdf 

    2. A patient of yours lives in a hostel and binges heavily on alcohol on weekends. On Monday morning staff at the hostel witness him having a seizure.
    Alcohol Withdrawal
    3. A woman of 55 who has drank a bottle of vodka every day for many years. She has severe problems with her memory and occasional hallucinations. She is disoriented to time.
    Wernicke’s encephalopathy
    EMI 8
    Case-control study
    Retrospective Cohort Study
    Cross sectional survey
    Ecological survey
    Open Label RCT
    Parallel RCT
    Cross over trial
    Qualitative study
    Intention to treat analysis
    Which would be the most ethical (choose ONE) for the following:
    1. To look at the number of suicides following an overdose.
    Retrospective Cohort

    2. To look at the number of children developing cardiac abnormalities in mothers on lithium
    Case-control study

    3. To look at obstetric complications and the development of schizophrenia but you wish to avoid attrition bias.
    Case-control study
    Ajetunmobi 59

    4. To look for exposure and outcome whilst avoiding recall bias
    Prospective Cohort Study
    Ajetunmobi 59

    5. To compare treatment vs placebo. The researchers wish to recruit fewer participants.
    Open Label RCT

    6. To compare whether a new drug has fewer side effects than another, established drug.
    Cross over trial
    Gosall and Gosall p. 17

    7. To get patient’s perspectives on the quality of care they receive.
    Qualitative Study
    Gosall and Gosall p. 12
      
    EMI 9
    Analysis of covariance
    Paired t test
    One sample T-Test
    Chi squared test
    ANOVA

    Choose ONE:

    1. Test to compare an antidepressant against placebo taking into account baseline differences.
    Analysis of covariance

    2. Test to compare recovery rates in antidepressant vs placebo at three centres. Data follows normal distribution.
    More than two groups, parametric data
    ANOVA
     
    3. Test to compare change in blood pressures in patients before and after a trial drug is given. Data is parametric and there is no control group.
    Same sample; parametric data,
    One Sample T-test
     
    EMI 10
    Options
    Buspirone
    Citalopram
    Desipramine
    Escitalopram
    Family Therapy
    Fluoxetine
    Haloperidol
    Lithium
    Logotherapy
    Methylphenidate
    Parental skills training
    Paroxetine
    Quetiapine
    Reboxetine
    Risperidone
    Sertraline
    Venlafaxine
    None of the above



    Lead in: Select one option each for the following

    Scenarios
    1: A 4 year old girl who is extremely aggressive towards her mother and has punched her in the stomach on one occasion. She hits other children at the nursery. She is ‘difficult’ and refused to do what she is told.
    Parental skills training

    2= A 17 year old boy with a history of frequent aggressive behaviour. Has been in trouble with the police. Parents got an injunction banning him from staying at home. He said he was aggressive almost all the time. Otherwise he has no symptoms.
    Risperidone (editted after discussion)


    3= A 13 year old girl who is aggressive and in trouble at school. She was diagnosed by psychologist to have ADHD. She is having multi-systematic therapy but is still hyperactive and has poor attention.
    Methylphenidate


    EMI 12
    • Terminally ill people are screened for depression with the question ‘Do you think you are depressed?’. This is then compared to a structured diagnostic interview used to diagnose depression. NB Calculators are not allowed.
    If you need more options, please add in red

    Options

    11%
    22%
    33%
    44%
    55%
    66%
    77%
    88%
    99%

    Note: gold standard at top
    Interview Depressed Not depressed
    Yes to question 11 9
    No to question 14 40
    Totals 25 49
    (1): What was the prevalence of depression in the sample?
    a+c
    a+b+c+d

    (2): What was the sensitivity of the screening?
    a
    a+c

    (3): What was the specificity of the screening?
    d
    b+d

    (4): What proportion of those who screened positive were depressed?
    Positive Predictive Value
    a
    a+b

    (5): What proportion of those who were depressed, screened positive?
    Again sensitivity
    a
    a+c

    6) What was the negative predictive value?
    d
    c+d
    Blog EntryAug 4, '08 10:08 AM
    for everyone

    Attachment: How to make a 2 x 2 table.doc

    See attached article for answers to below mentioned exercises. (Exercise scenarios are  not real and are made up for learning purposes)
    Please correct any mistakes.
    Questions in exercises:

    Exercise 1:
    40 out of 50 people working in a factory exposed to grasbestos (excuse my neologisms) developed pneumoconiosis from 1930 to 1940. In a group of healthy football players (n = 37) only one person developed lung disease.
    Draw a 2 x 2 table
    Calculate EER, CER, Risk Ratio, odds for exposure group, odds for control group, Odds Ratio, absolute risk reduction, relative risk reduction and number needed to harm.

    Exercise 2:
    A cohort of 700 men was randomised for the trial of a new antipsychotic salanzapine (name made up for learning purposes). 380 people received the new drug while the rest received risperidone.
    300 of the intervention group showed significant improvement in PANSS scores while the number of responders for risperidone group was 280.
    Draw a 2 x 2 table.
    Calculate EER, CER, Risk Ratio, odds for exposure group, odds for control group, Odds Ratio, absolute benefit increase, relative benefit increase and number needed to treat.
    EXERCISE 3:
    A psychiatrist in Dublin, made a new depression rating scale called DMI (Dublin Mood Inventory) (name made up for learning purposes)
    40 out of 50 subjects from a hospital setting were diagnosed to be depressed on BMI. 45 subjects were found depressed on Hamilton Depression Rating  Scale including 39 subjects who were also found depressed on the (so called) DMI.
    Make a 2 x 2 table, calculate sensitivity, specificity, PPV, NPV, likelihood ratio for a positive result,  likelihood ratio for a negative result, pre-test probability and  post-test probability for this instrument.

    SUGGEST MORE AND MORE SCENARIOS LIKE THIS.

    Attachment: How to make a 2 x 2 table.doc
    Blog EntryMay 29, '08 7:58 PM
    for everyone

    2 MRCPsych 2008 onwards / CASC / Re: Activities of Daily Living
     on: 17th Sep 2006 at 20:35 



    Activities of Daily Living
    Mnemonic 
    BASICS

    asic abilities
    Advanced abilities
    Safety
    Issues and risks
    Compensatory strategies
    Social Abilities


    Basic Abilities: include toiletting, dressing , self care and eating. (Personal ADLs)

    Advanced abilities include : (mostly domestic and external ADLs)
    Shopping, cooking and handling money

    Safety Issues include : Gardening, use of electricity, use of gas, vulnerability, driving and awareness and care for security needs. (mostly domestic and external ADLs)

    Compensatory Strategies include: Notes, diaries,  mobile phone use and  reminders etc.

    Social Abilities: include parenting, maintaining relationships and  socializing with friends (Social ADLs)
    Blog EntryMay 28, '08 8:33 AM
    for everyone

    http://www.mrcpsychforum.co.uk/1.html


    ANSWERS*
    *  = answers by MRCPsych forum admin.
    *  = answers modified by Kamran     
    1. Withdrawal symptoms occur after how many hours of stopping
    smoking?
    a. 2-3 hours *
    b. 4-6 hours
    c. 8-12 hours *
    d. 12-14 hours
    e. > 24 hours

    2. An evaluation of a new screening questionnaire for depression in
    primary care is conducted. The most important single feature of this
    questionnaire that would encourage you to use it is:
    a. It has Positive Predictive Value of 20%
    b. It has sensitivity of 80% *
    c. It has specificity of 60%
    d. It is brief
    e. It is validated in different languages

    3. Which of the following clinical questions is better suited to qualitative
    rather than quantitative research methods?
    a. A study detailing the social outcomes of people treated with Cognitive
    Behavioural Therapy
    b. A study of the costs per unit of ‘patient utility’ in a randomized
    controlled trial
    c. A study to assess the attitudes of people with borderline personality
    disorder to their diagnosis *
    d. A study to measure psychiatric symptoms after head injury
    e. A study to measure quality of life in people with Alzheimer’s disease.

    4. Which of the following is the least adequate method of
    randomisation?
    a. Minimisation
    b. Odd/even last digit of date of birth *
    c. Permuted block randomisation
    d. Simple randomisation by computer
    e. Toss of a fair, unbiased coin.

    5. The most important component in the development of a clinical
    guideline is:
    a. All stakeholders are consulted.
    b. Economic factors are considered
    c. Meta-analysis of the available data *


    e. Most hypomanic episodes in bipolar II disorder last at least 4 days.


    10. In comparing bipolar depression with unipolar depression all are true
    except:
    a. There are significant phenomenological differences
    b. Atypical symptoms are more common in unipolar depression *
    c. Where hypomania occurs as an adverse event in the treatment of
    unipolar depression, this probably means the patient has bipolar
    disorder
    d. Psychotic depression in earlier adulthood is strongly suggestive of
    bipolar disorder
    e. A family history of bipolar disorder is more common in bipolar
    depression.


    11. The following may make the diagnosis of hypomania difficult except:
    a. Recent antidepressant therapy
    b. Substance misuse
    c. Borderline personality disorder
    d. Symptoms of attention-deficit hyperactivity disorder
    e. Hypothyroidism *


    12. Which of the following is not true in Parkinson’s disease dementia:
    a. Is uncommon *
    b. Is difficult to distinguish from Lewy-body dementia
    c. Results in deficits in executive dysfunction
    d. Presents with hallucinations more frequently than does Alzheimer’s
    disease
    e. Patients are sensitive to antipsychotics.



    13. As regards movement disorders in Parkinson’s disease all are true
    except:
    a. Medication may affect psychiatric symptoms
    b. They may be aggravated by psychotropic medication
    c. Peak plasma doses of levodopa always trigger psychotic symptoms *
    d. They may mask depression
    e. Presentation together with cognitive impairment has a poor prognosis.




    14. Regarding psychiatric symptoms in Parkinson’s disease dementia:
    a. There is good evidence for antipsychotic efficacy
    b. Depression is straightforward to diagnose
    c. Phenomenologically, psychotic symptoms are indistinguishable between
    Parkinson’s disease dementia and Lewy-body dementia, but the
    frequency of occurrence is greatest in the latter *
    d. They should be considered in isolation from motor symptoms
    e. Hallucinations are most commonly tactile.



    15. Cholinesterase inhibitors in Parkinson’s disease dementia:
    a. Are well tolerated
    b. Show some efficacy for psychotic symptoms *
    c. Have established benefits for behavioural symptoms
    d. Offer no benefit for cognitive symptoms
    e. Assist in motor symptom control.



    16. In Parkinson’s disease dementia:
    a. Hallucinations are rarer than in Parkinson’s disease without dementia
    b. Psychotic symptoms are predictive of the need for nursing home
    placement *
    c. Depression may be more severe than in Parkinson’s disease with
    dementia
    d. MRI is useful in routine clinical practice to distinguish from Lewy-body
    dementia
    e. Depression is identical to depression in people without Parkinson’s
    disease dementia.



    17. A 15-year-old girl presenting to a hospital A&E after an overdose:
    a. Should be sent home after triage by A&E staff
    b. Is unlikely to be at risk of later suicide
    c. May benefit from family therapy *
    d. Is more likely than not to benefit from antidepressant medication.
    e. Should always be offered debriefing



    18. A 15-year-old in the care of the local authority hangs himself 1 week
    after discharge from a psychiatric unit. The incident:
    a. Should be the subject of a Section 15 inquiry
    b. Should be reported to the National Patient Safety Agency *
    c. Should be reported to the GMC
    d. Will not be looked at from the point of view of the ‘Safety First’
    recommendations.
    e. Should lead to dismissal of the CAMHS consultant



    19. Common adverse incidents in CAMHS include:
    a. Homicide
    b. Deliberate self-harm *
    c. Assaults on fellow in-patients
    d. Medication errors
    e. Discharge against medical advice



    20. Child psychiatrists:
    a. Should always share information freely
    b. Need not be concerned with risks presented to others if there is no
    formal mental illness
    c. Should discuss confidentiality issues with the Caldicott guardian *
    d. Should be able to defend their case management actions with evidence
    from systematic reviews
    e. Should always follow Pritchard criteria


    21. All are true for dissociative disorders of childhood and adolescence
    except:
    a. Of both transient and chronic type occur more commonly in girls *
    b. When transient, often respond well to reassurance alone
    c. When prolonged beyond a few weeks, usually share many
    characteristics with other ‘somatising’ disorders
    d. Are associated with school presentations and learning problems
    e. Can be associated with stress event



    22. The development of somatising disorders during childhood
    demonstrates that:
    a. Persistent somatoform pain disorder is developmentally the last to
    appear
    b. Somatising disorders may be preceded by adjustment reactions with
    prominent physical symptoms *
    c. Characteristic presenting complaints for ICD– 10 somatoform disorders
    are very similar in children and adults
    d. It is a good predictor for later life Hypochondriasis
    e. Hypochondriacal disorder is not unusual in middle childhood.



    23. An increased likelihood of reporting somatic symptoms in childhood
    (pre-adolescence) is associated with all except:
    a. difficult behaviour
    b. negative-impact life events
    c. anxiety traits
    d. conscientious and perfectionist temperamental traits
    e. Above average school achievements in most cases. *



    24. Factitious disorders in childhood and adolescence:
    a. are almost always associated with severe child psychopathology
    b. are not characterised by parents' fabrication of symptoms in
    adolescents
    c. are best assessed as a form of conversion disorder
    d. present commonly to ophthalmologists and dermatologists *
    e. Can commonly present in conjunction with serious physical illness



    25. With regard to Clozapine all are true except:
    a. response to treatment with Clozapine is subject to interindividual
    variations
    b. it is primarily metabolised by CYP1A2
    c. it is unlikely to be metabolised by CYP2D6
    d. dopamine D4 receptor polymorphisms have been shown to be strongly
    associated with Clozapine response *
    e. Combinations of pharmacodynamic gene polymorphisms have been
    shown to predict response to Clozapine


    26. Regarding tardive dyskinesia all are true except:
    a. Susceptibility to the development of tardive dyskinesia is thought to
    have a genetic basis
    b. The AIMS is a commonly used scale for rating the presence of tardive
    dyskinesia
    c. The DRD3 Ser9Gly polymorphism has been significantly correlated with
    the development of tardive dyskinesia
    d. The development of tardive dyskinesia has been linked with CYP1A2
    e. No association has been shown between the development of tardive
    dyskinesia and superoxide dismutase genes *



    27. The serotonin transporter-linked polymorphic region (5-HTTLPR):
    a. Is not an insertion/deletion polymorphism
    b. Has been identified in intron 2 of the serotonin transporter gene
    c. Has not been associated with clinical response to SSRIs
    d. Has not been associated with clinical response to TCAs
    e. Has been associated with response to lithium treatment *



    28. The following are genes thought to be associated with the
    corresponding phenotypes except:
    a. The 5-HT2A gene and response to treatment with electroconvulsive
    therapy
    b. The serotonin transporter gene family and Clozapine-induced
    agranulocytosis *
    c. The phospholipase C {gamma}-1 gene and lithium response in the
    treatment of bipolar disorder
    d. The apolipoprotein {varepsilon}4 gene and poor response to tacrine in
    the treatment of Alzheimer’s disease
    e. The dopamine transporter gene and response to methylphenidate in
    the treatment of ADHD



    29. Which of the following statements is true about drug metabolism?
    a. Haloperidol is metabolised via CYP2D6 *
    b. CYP2D6 polymorphisms do not affect the metabolism of nortriptyline
    c. Poor CYP2D6 metabolisers have multiple functional copies of a CYP2D6
    gene
    d. Up to 10% of Caucasians may be CYP2D6 ultrarapid metabolisers
    e. The P450 enzymes are Phase II drug-metabolising enzymes



    30. As regards identification and meaning of explanatory models:
    a. Only anthropologists can elicit explanatory models
    b. Participant observation is essential for identifying the explanatory
    models of patients from other cultures
    c. Explanatory models are cognitive representations of insight-related
    beliefs
    d. Explanatory models are delusional, but culturally determined delusions
    e. Talking to the patient’s family and peer group can help unravel the
    cultural origins of their explanatory models. *



    31. As regards explanatory models in the clinical consultation:
    a. Explanatory models can be treated with neuroleptics
    b. Explanatory models are stable representations of a patient’s world view
    about their problems
    c. Negotiation of different explanatory models is key to improved
    communication *
    d. A multi-lingual interpreter should be able to relay a complete picture of
    a patient’s world view
    e. Patients who replace their own explanatory model with that of the
    psychiatrist are likely to be totally compliant with the psychiatrist’s
    recommendations.



    32. The following factors commonly influence the basic features of
    explanatory models except:
    a. Age
    b. Gender
    c. History of good physical health *
    d. Educational status
    e. Time-point and context of assessment.



    33. Cultural formulation includes all except:
    a. The therapist’s view of the patient’s cultural identity *
    b. Cultural explanations of distress
    c. The influence of the psychosocial environment
    d. Cultural elements in the patient–therapist relationship
    e. Noting the patient’s culture in reaching a diagnosis.



    34. Theoretical sampling is based on:
    a. Convenience
    b. Developing a theory *
    c. Randomisation
    d. Intuition
    e. Non-emerging categories


    35. Qualitative methods are useful in all except:
    a. Hypothesis generation
    b. Treatment evaluation *
    c. Investigating anomalous results
    d. Exploring barriers to policy implementation
    e. Finding out user views.



    36. Qualitative methods include all except:
    a. ethography
    b. focus groups
    c. participant observation
    d. case–control study *
    e. semi-structured interviews



    37. To demonstrate validity a qualitative study should be:
    a. implicit
    b. esoteric
    c. non systematic
    d. exclusive
    e. Reflexive. *



    38. Reflexivity involves:
    a. consideration of the impact of the presence of the disease on data
    collected
    b. awareness of the patient as a positioned observer in research
    c. achieving the lotus position
    d. recognition of the researcher's preconceptions and their influence on
    interpretation and analysis *
    e. knowledge of the works of Karl Marx.


    39. Studies concerning supervised discharge and similar legislation have
    found:
    a. it most effective for those with non-affective psychosis *
    b. frequent contact with services is unnecessary
    c. no reductions in hospitalisation
    d. use of existing powers is widespread and uniform
    e. no suggestion of reductions in violence.


    40. With regard to restriction orders:
    a. they can only be made after conviction for a serious offence
    b. they allow patients to be discharged only by the Home Secretary
    c. conditionally discharged patients are less likely to be convicted of an
    offence than prisoners released on life licence
    d. they allow conditionally discharged patients to be recalled to hospital
    at any time by their responsible medical officer
    e. they allow conditionally discharged patients to be readmitted to
    hospital informally without being recalled. *


    41. Guardianship and supervised discharge:
    a. both require patients to reside where directed, allow access to mental
    health professionals, and attend for treatment *
    b. are both ‘health-led’ and administered by medical records departments
    c. patients on both can be ‘taken and conveyed’ to hospital
    d. have effective sanctions in the face of non-compliance
    e. have both been subject to extensive research.



    42. All are true about probation orders with a condition of psychiatric
    treatment except:
    a. are rarely made
    b. can only be made if arrangements have been made for treatment
    c. in studies have been successful in 60% of cases
    d. are made for the majority of mentally disordered offenders subject to
    probation supervision *
    e. have had statutory recognition for over 50 years.



    43. Methodological factors that affect results of prevalence studies of
    mental health problems in people with learning disability include all
    except:
    a. The definition of learning disability used
    b. The definition of mental health used
    c. The diagnostic criteria used
    d. Not using mental impairment as defined in mental health act 1983 *
    e. The method used for the identification and assessment of cases.
    44. The total prevalence rate (including problem behaviour) of mental
    health problems in adults with learning disability is probably
    between:
    a. 5 and 25%
    b. 50 and 80%
    c. 30 and 50% *
    d. 5 and 10%
    e. 10 and 60%.



    45. All are true about life events in adults with learning disability except:
    a. occur at a rate similar to that in the general population
    b. have been associated with depressive symptoms
    c. have been associated with anxiety and depressive symptoms
    d. have been associated with psychotic disorders *
    e. require further research to determine their role in the aetiology of
    mental health problems.



    46. The most common forms of problem behaviour in learning disability
    include all except:
    a. Self-injury
    b. Stripping *
    c. Verbal aggression
    d. Demanding
    e. Physical aggression


    47. The most common compulsion found in people with learning
    disability is:
    a. hand-washing
    b. checking
    c. ordering *
    d. cleanliness
    e. touching.


    48. As regards PTSD:
    a. it is overdiagnosed
    b. resilience is not a dynamic quality
    c. prevalence of PTSD is the same all over the world
    d. prior trauma does not increase susceptibility to PTSD
    e. chronic pain is associated with PTSD. *


    49. In PTSD:
    a. avoidance is associated with chronicity *
    b. refugees are at no more risk than the indigenous population
    c. misdiagnosis as refractory depression is not a problem
    d. cortisol levels are high
    e. securely attached individuals exhibit more symptoms.


    50. In the aetiology of PTSD:
    a. the hippocampus has no role
    b. locus of control has no role
    c. the sympathetic nervous system is not involved
    d. lower level of education is protective
    e. unresolved childhood trauma increases the risk. *

    51. Vulnerability factors for PTSD include:
    a. male gender
    b. middle age
    c. internal locus of control
    d. good impulse control
    e. Personality disorders. *


    52. Defence mechanisms used to deal with trauma include:
    a. identification
    b. denial *
    c. rationalisation
    d. sublimation
    e. splitting.


    53. Concerning coroners in England and Wales:
    a. they must hold medical and legal qualifications
    b. the majority are full-time
    c. they can be dismissed only by the Home Secretary
    d. they can appoint their own deputy *
    e. there are 129 coroners in England and Wales.


    54. Concerning the role of the coroner:
    a. they must investigate all reported deaths
    b. they must decide whether a post-mortem is necessary *
    c. they are obliged to inform the media about an inquest
    d. they record data for the district courts
    e. they are not permitted to question the witnesses.
    55. Regarding an inquest all are true except:
    a. the process is inquisitorial
    b. it is held in the Crown Court *
    c. the intention is not to identify those who are at fault
    d. members of the deceased’s family can question witnesses
    e. a jury sits in a minority of cases.


    56. Regarding giving evidence to an inquest all are true except:
    a. witnesses are under oath
    b. witnesses can request anonymity
    c. witnesses can refuse to answer questions that may incriminate them
    d. an ordinary witness should offer an opinion on the facts of the case *
    e. witnesses should not use technical language wherever possible.


    57. The following have clearly been shown to be risk factors for
    Alzheimer's disease:
    a. male gender
    b. head injury
    c. aluminium
    d. age *
    e. maternal age.


    58. Regarding Alzheimer's disease:
    a. oestrogen therapy has recently been shown to improve cognition and
    disease progression
    b. vitamin E has recently been shown to improve cognition
    c. insulin-dependent diabetes has been shown to be an independent risk
    factor *
    d. treatment of isolated systolic hypertension has not been found to
    reduce incidence
    e. non-steroidal anti-inflammatory drugs are assoicated with risk of
    Alzheimer's disease.


    59. Regarding dementia all are true except:
    a. Alzheimer's disease is more common than vascular disease in Asian
    countries *
    b. the survival rate in vascular dementia is lower than in Alzheimer's
    disease
    c. 20% of patients with Alzheimer's disease have cerebral vascular
    lesions
    d. more than 25% of patients may develop dementia following a stroke
    e. Elevated blood pressure early in life is a risk factor for dementia.


    60. Accident and emergency:
    a. attendances resulting in admission to a ward have increased as a
    result of reduced hospital bed numbers
    b. staff should avoid any assessment of patients’ mental state
    c. staff should carry out an assessment of risk factors for further selfharm
    *
    d. triage protocols reliably give appropriate importance to mental health
    problems
    e. 4 h waiting-time limits are always seen as beneficial to patients who
    have self-injured.

    61. Considering self-injury patients on surgical wards:
    a. all surgical wards have extensive experience of patients who self-harm
    b. the restrictions placed on patients who have self-injured are always
    consistent with the level of immobility caused by their injuries
    c. there is no evidence that patients who self-injure receive less frequent
    analgesia
    d. opiate users require less analgesia following self-injury, as they
    already have sufficient levels of opiate in their system
    e. Patients’ low self-esteem may be unintentionally exacerbated by the
    normal routine of the staff. *


    62. Patients who attend hospital frequently following self-injury:
    a. are generally viewed sympathetically and paternalistically by hospital
    staff
    b. always, by virtue of familiarity, receive a more thorough assessment
    c. always require enhanced CPA
    d. should have a coordinated care plan identifying the kind of assessment
    that should occur at future attendances *
    e. Have complex needs that make any psychosocial assessment by acute
    hospital staff unnecessary.


    63. The management of self-harm involves:
    a. risk assessment as an academic fact-finding exercise
    b. a change from risk assessment to the new concept of psychosocial
    assessment, which ignores the formulation of a patient’s psychological
    and social problems
    c. lack of acknowledgement of the cause of harm in order to reduce
    stress in the patient
    d. a psychosocial assessment that accurately predicts the risk of
    repetition
    e. a management plan informed by thorough psychosocial assessment. *


    64. The process of changing attitudes to self-injury may be aided by:
    a. improving training for medical and surgical staff only
    b. involvement of service users in staff training *
    c. ensuring that acute hospitals and mental health staff work entirely
    separately of each other
    d. the continued view that the needs of people who self-harm are
    someone else’s problem
    e. The tendency to blame other parts of the service for perceived
    deficiencies.



    65. Which of the following is false about “Denial”:
    a. Must always be confronted *
    b. Can not be reduced with antidepressants
    c. May be a fluctuating process
    d. In some patients remains until death
    e. Should be confronted only if it causes significant problems for the
    patient or family.


    66. Depressive disorders in terminal illness:
    a. are uncommon in palliative care settings
    b. are generally picked up
    c. may be diagnosed from somatic symptoms
    d. do not respond to antidepressants
    e. may respond to amphetamines. *


    67. All are true about Delirium in palliative care except:
    a. occurs in over 50% of patients at some stage in their illness
    b. may present with sedation
    c. always responds to neuroleptic medication *
    d. may be irreversible
    e. May be helped by haloperidol.


    68. As regards drug therapy in palliative care all are true except:
    a. lorazepam is useful for acute anxiety
    b. amitryptyline is useful for neuropathic pain
    c. risperidone may be useful for psychotic symptoms in terminally ill
    patients
    d. methotrimeprazine reduces agitation
    e. selective serotonin reuptake inhibitors are more effective than tricyclics
    in the management of depression in terminal illness *


    69. All are true about Cognitive–behavioural therapy except
    a. can help patients with body image disturbance
    b. is not useful for pain *
    c. can be used at the same time as medication
    d. can improve hopelessness in terminally ill patients
    e. can help to reduce symptoms of depression in terminal illness.


    70. Regarding affective disorder in renal patients all are true except:
    a. major depressive disorder tends to occur more often than adjustment
    disorder *
    b. adjustment disorder can lead to behavioural problems.
    c. adjustment disorder is likely to influence physical outcome.
    d. symptoms less indicative in the diagnosis of major depressive disorder
    are lack of energy and insomnia.
    e. adjustment disorder may become a chronic problem.


    71. Cognitive therapy:
    a. has little evidence base in liaison settings
    b. can not be used to modify negative automatic thoughts about dialysis
    c. may help patients to regain a perception of control in their illness *
    d. will not teach techniques that a patient may use to deal with future
    problems
    e. is directed by the therapist's perception of the patient's problem.


    72. The following substances are associated with violence in nonpsychotic
    populations except:
    a. alcohol
    b. opiates *
    c. amphetamines
    d. cannabis
    e. cocaine


    73. Men who misuse alcohol and have a diagnosis of schizophrenia:
    a. present a similar risk of violence as men with schizophrenia without
    alcohol misuse
    b. present with a much higher risk of violence than individuals with
    personality disorder and alcohol misuse
    c. commonly present with medication non- compliance *
    d. should not be diagnosed as suffering from a dependence syndrome if
    they meet the criteria for a diagnosis of schizophrenia
    e. Have more incidence of Clozapine refractory schizophrenia


    74. The following criteria must be considered for an ICD-10 diagnosis of
    a dependence syndrome:
    a. compulsion to take the substance *
    b. craving
    c. increased tolerance
    d. offending
    e. depression


    75. For offenders with a history of alcohol misuse and violent offending,
    therapeutic approaches in prison can include all except:
    a. an assessment by a specialist substance misuse service
    b. involvement of voluntary sector counselling services
    c. sulfiram *
    d. detoxification
    e. risk assessment


    76. Factors to consider in risk assessment of violent offenders with a
    diagnosis of schizophrenia and substance misuse are all except:
    a. index offence
    b. post-offending history
    c. insight
    d. number of first rank symptoms *
    e. treatment compliance.


    77. All are true about depression in multiple sclerosis except:
    a. has a lifetime prevalence of up to 50%
    b. can be caused by the drugs used to treat the multiple sclerosis
    c. can not be treated with SSRIs *
    d. can be treated with cognitive–behavioural therapy
    e. can be treated with ECT, but only with caution.


    78. Cognitive impairment in multiple sclerosis:
    a. is no more common than in the general population
    b. can be explained by the increased prevalence of depression in multiple
    sclerosis
    c. is best screened for using the Mini-Mental State Examination
    d. can not be treated with donepezil
    e. has the same features as a subcortical dementia. *

    79. Delirium:
    a. typically includes a broad range of neuropsychiatric symptoms *
    b. has disturbances of memory as a central feature
    c. can be diagnosed by the presence of clouding of consciousness alone
    d. rarely involves mood disturbances
    e. is a subtype of acute confusion.

    80. In the detection of delirium:
    a. non-identification is rarely a problem in clinical practice
    b. up to two-thirds of cases are superimposed on dementia *
    c. detection can be erroneously enhanced by routine cognitive
    assessment of all patients
    d. prevalence has decreased in recent years
    e. Misdiagnosis as dementia is rare


    81. Hypoactive or quiet delirium:
    a. is frequently missed in clinical practice *
    b. has a better prognosis than agitated or hyperactive delirium
    c. does not respond to antipsychotic agents
    d. frequently makes patients unrousable
    e. rarely includes delusions or hallucinations.


    82. In the management of patients with delirium:
    a. iatrogenic causes of delirium are common *
    b. involvement of relatives is generally discouraged
    c. risk factor reduction does not allow episode prevention
    d. delirious patients should not contribute to treatment decisions
    e. the effectiveness of antipsychotics is principally due to sedative
    actions.


    83. In delirium assessment:
    a. delirium rating scales allow distinction of delirium from dementia *
    b. delirium cannot be accurately assessed in mute patients
    c. the CAM has good coverage of delirium symptoms
    d. the DRS has poor coverage of delirium symptoms
    e. the MMSE has good coverage of delirium symptoms.


    84. Likely risks of TMS include:
    a. interference with hormonal implants
    b. induction of excitotoxicity
    c. memory loss
    d. complications of the general anaesthetic
    e. induction of seizures in predisposed patients. *


    85. In depression:
    a. TMS can be used to investigate brain mechanisms of the illness *
    b. there is overall no difference between active and sham treatments
    c. it is completely impossible for patients to distinguish sham and active
    treatment
    d. TMS is as effective as ECT in psychotic depression
    e. TMS's positive effects on mood are outweighed by its negative effects
    on memory.


    86. In traumatic brain injury:
    a. The best predictor of outcome is the period of post-trauma amnesia *
    b. late-onset agitation predicts long-term aggression
    c. aggression and agitation respond differently to medication
    d. suicidality has no association with hopelessness
    e. There is no evidence of an association with schizophrenia.


    87. Suicide rates have been shown to be higher in people with all except:
    a. Huntington’s disease
    b. hedonistic homoeostatic dysregulation *
    c. traumatic brain injury
    d. Akathisia
    e. stroke


    88. Research and investigations into sleep disorders have suggested
    that:
    a. hypocretins can be differentiated from orexins as having a role in sleep
    and arousal
    b. there is an HLA association with REM sleep behaviour disorder
    c. sleep latency of more than 10 min on overnight polysomnography is
    characteristic of narcolepsy
    d. clonazepam is the first-line treatment for narcolepsy
    e. REM sleep behaviour disorder can precede future movement disorders.
    *


    89. In Parkinson’s disease:
    a. Psychosis is associated with decreases in antiparkinsonian medications
    b. Sulpiride is an appropriate treatment of psychotic symptoms
    c. Hypersexuality, hypomania and pathological gambling can be seen with
    increased doses of dopamine-replacement medications *
    d. Dementia should be treated with rivastigmine as there is randomised
    controlled trial evidence in favour of this
    e. Tricyclic antidepressants worsen motor symptoms.



    90. People with epilepsy:
    a. Should not be given psychotropic medications as they increase mean
    seizure frequency
    b. When treated with carbamazepine can be given fluoxetine without
    monitoring of serum levels
    c. Can also have non-epileptic seizures *
    d. As well as non-epileptic seizures need a lumbar puncture to ascertain
    prolactin levels for diagnostic purposes
    e. Taking tricyclics may be more likely to experience increase in seizure
    threshold than those taking SSRIs.


    91. The incidence of treatment-resistant schizophrenia is:
    a. 5%
    b. 10%
    c. 20% *
    d. 50%
    e. 75%.


    92. The most commonly used categorical definition of treatment
    resistance is :
    a. prolonged hospitalisation
    b. less than a 20% improvement on the BPRS *
    c. symptoms enduring for more than 2 years
    d. persistent negative symptoms
    e. Persistent cognitive symptoms.


    93. Definitions of treatment resistance in Schizophrenia might also
    include except:
    a. non-adherence
    b. failure of maintenance
    c. comorbidity
    d. aggression *
    e. residual symptoms


    94. The following statements about selective serotonin reuptake
    inhibitors are correct except:
    a. Fluoxetine in indicated during breast feeding *
    b. They are less sedating than tricyclic antidepressants
    c. Sertraline causes a dry mouth
    d. Fluoxetine causes anorexia
    e. Fluvoxamine causes headaches


    95. Excessive fluid intake in psychiatric patients:
    a. Will not produce acute psychiatric symptoms when the plasma sodium
    is around 140 mmol/l *
    b. Should be treated with hypertonic saline if intoxication occurs
    c. It is most commonly found to be a compulsive activity
    d. Beyond 2.5% of basal body weight renders intoxication likely
    e. Is reliably detected by an early-morning plasma sodium estimation



    96. A moderating variable in a treatment trial:
    a. Refers to the quality of the treatment manual
    b. Is a variable altered by the treatment that affects outcome
    c. Is a variable independent of treatment which alters the effect of
    treatment on outcome *
    d. Is a variable measured after the treatment as an indicator of how the
    treatment has moderated the local environment
    e. Is a factor that always makes the treatment effect smaller than it
    would have been otherwise.



    97. As regards the therapeutic alliance:
    a. It has no association with pre-treatment variables
    b. It has been shown to vary with latent symptom change late in
    treatment
    c. It has been shown to have an impact in both psychological and drug
    treatments *
    d. It is an example of a moderating variable as strictly defined
    e. Its positive effects are most clearly seen in the treatment of
    internalising disorders in children.


    98. As regards testing for statistical mediation, all are true except:
    a. A proposed mediator must not show an independent effect on outcome
    *
    b. A mediator reduces the measured effect of treatment on outcome in a
    regression analysis
    c. A mediating variable shows an association with treatment effect
    d. It can even be done in studies other than randomised controlled trials
    e. It gives us information about how a treatment has its effect.


    99. As regards pragmatic trials:
    a. it is essential to have modelled all the details of the intervention
    beforehand
    b. the main aim is to tell us how a treatment works
    c. they need to have high external validity *
    d. they are generally less valid than explanatory trials
    e. they have more exclusive referral criteria than explanatory trials.


    100. As regards randomisation:
    a. it can only be performed at the level of individual patients
    b. it guarantees that the study will be free of confounders
    c. it automatically leads to poor external validity (or generalisability)
    d. concealment of allocation of randomisation refers to whether the
    outcome of randomisation can be predicted *
    e. non-randomised studies are less likely to be affected by confounding
    than randomised studies.


    101. The outcomes used in pragmatic RCTs can be all of the following
    except:
    a. Functional outcomes should be emphasised
    b. are assessed blind, to prevent observer bias
    c. might include cost-effectiveness of interventions
    d. aim to be short-term outcomes *
    e. May include ‘process outcomes’.


    102. All are true regarding complex treatments (such as
    psychotherapies) except:
    a. might be properly tested in randomised trials
    b. are difficult to give blind
    c. may have differential drop-outs between groups because of the
    patients’ preferences
    d. cannot provide realistic control conditions for such treatments *
    e. May have limited generalisability of results because of patient
    selection.


    103. What is the most common MHA section throughout the country?
    a. Section 2
    b. Section 5(2)
    c. Section 3 *
    d. Section 37
    e. Section 17


    104. What is the maximum number of categories of mental disorder
    that can be applied to a person detained under section 3?
    a. One
    b. Two
    c. Three *
    d. Four
    e. Five


    105. What does ASW stands for?
    a. Annoyed Social Worker
    b. Angst-ridden Social Worker
    c. Approved Social Worker *
    d. Accredited Social Worker
    e. Assistant Social Worker



    106. What are the age limits for detaining someone?
    a. No age limits for either old or young *
    b. No age limits for old but more than 14 for young
    c. No age limits for young but less than 75 for old
    d. No age limits for old but more than 16 for young
    e. More than 14 for young and less than 75 for old


    107. When a section 3 is renewed for the first time and the patient did
    not appeal, which of these safeguards does not apply?
    a. Patient referred for a Tribunal if they had not appealed during the first
    six months
    b. If the patient was not consenting to treatment a second opinion is
    always arranged *
    c. The hospital managers review the detention
    d. Information about the renewal of detention is always given to the
    patient by a nurse
    e. Their needs are taken fully into account



    108. On what grounds can a consultant bar discharge by a nearest
    relative?
    a. Imminent risk to public
    b. Significant deterioration of mental health will happen
    c. Self-care of client out of hospital will be a problem
    d. Likely dangerousness to self or others *
    e. Aftercare has not been arranged



    109. New mental health act contains only one category of:
    a. Mental impairment
    b. Mental incapacity
    c. Mental disorder *
    d. Mental illness
    e. Mental disability
    110. Which of the following does not constitute “appropriate treatment”
    under new mental health act?
    a. Intramuscular depot injection
    b. Oral Risperidone
    c. Seclusion
    d. ECT
    e. Timeout *


    111. Following drugs have been shown to have some evidence for
    treatment of psychosis in mild to moderate Huntington’s disease:
    a. Haloperidol *
    b. Aripiprazole
    c. Ziprasidone
    d. Olanzapine
    e. Sulpiride



    112. Evidence has shown that patient’s compliance with medication is
    most affected by:
    a. Side effects
    b. Insight
    c. Treatment effectiveness *
    d. Carer support
    e. Specialist treatment



    113. Failure to convulse with ECT can be associated with all of the
    following except:
    a. narrow separation of the electrodes
    b. abnormal skull thickness
    c. unilateral ECT
    d. use of methohexitone anaesthetic
    e. use of a modified sine-wave stimulus *


    114. All are true about cannabis except:
    a. causes a dry mouth
    b. causes insomnia in withdrawal after chronic use
    c. is unable to enter breast milk *
    d. potentiates warfarin
    e. impairs reproductive function



    115. Efavirenz is associated with all except:
    a. twilight states
    b. Suicides *
    c. personality changes
    d. major depression and severe suicidal ideation
    e. psychosis


    116. All of the following are true regarding HIV infection and its
    treatment except:
    a. patients receiving antiretrovirals cannot safely use St John’s wort
    b. patients on antiretrovirals have an increased risk of developing
    extrapyramidal side-effects when exposed to neuroleptics
    c. SSRIs are generally well tolerated by patients receiving antiretrovirals,
    although there are pharmacokinetic interactions between some of
    these medications
    d. clozapine is safe in patients receiving protease inhibitors *
    e. there are significant interactions between antiretrovirals and
    recreational drugs.





    117. Regarding Parkinson’s disease and its treatment all are true
    except:
    a. antiparkinsonian medication can induce psychotic symptoms
    b. mood disorders are commonly seen in patients with Parkinson’s
    disease
    c. treatment of psychotic symptoms in Parkinson’s disease includes
    adjustment of the daily dose of antiparkinsonian medication
    d. psychotic symptoms in patients with Lewy body dementia can be safely
    treated with neuroleptics *
    e. combining SSRIs or tricyclics and selegiline can precipitate a serotonin
    syndrome.



    118. Regarding epilepsy and its treatment all are true except:
    a. psychotic symptoms are common in patients with epilepsy
    b. all anti-epileptics have been associated with development of psychotic
    symptoms
    c. withdrawal of antiepiletics may cause psychosis
    d. TCAs are safe in epileptic patients and are the first line of treatment for
    depression in epilepsy *
    e. lithium does increase seizure severity.



    119. The following statements are correct except:
    a. non-steroidal anti-inflammatories may induce psychiatric symptoms
    b. antihistamines may induce psychosis and delirium
    c. diuretics are not associated with development of psychiatric sideeffects
    *
    d. antibiotics can cause delirium
    e. antibacterials can induce depression



    120. Which one of the following is an incorrect statement about ICD–
    10:
    a. The application of diagnostic criteria of most disorders requires an
    understanding of their cause
    b. The multi-axial classification system for childhood disorders has five
    axes *
    c. Keeping to one psychiatric diagnosis where at all possible is
    encouraged
    d. The multi-axial classification system for childhood disorders includes an
    axis for family history of psychiatric disorder
    e. Subtypes of anxiety can be distinguished with a high degree of
    reliability



    121. In hepatic encephalopathy all are true except
    a. the EEG changes correlate with the degree of neuropsychiatric
    disturbance
    b. treatment should include a protein rich diet *
    c. hypersomnia is a typical feature
    d. personality change is associated with chronicity
    e. chronic irreversible dementia can develop



    122. The following are classical psychoanalytical concepts elaborated
    by Sigmund Freud except:
    a. repression
    b. isolation *
    c. reaction formation
    d. sublimation
    e. rationalization


    123. In the context of an elderly person, an appointee is:
    a. Officially designated by the Court of Protection
    b. Not designated for a patient in hospital
    c. The same as an agent
    d. Unable to send money, such as social security, collected for the elderly
    person
    e. Not designated if receivership is indicated *


    124. All are true about Borderline personality disorder except
    a. Responds in many cases to low-dose neuroleptic treatment
    b. Has origins in the work of Kernberg
    c. Includes the characteristic of feeling persistently bored
    d. Is a diagnosis which has poor stability over time *
    e. Is subsumed under emotionally unstable personality disorder in ICD 10


    125. With respect to the assessment of attachment:
    a. Ainsworth devised the Relationship Scales Questionnaire
    b. the Relationship Scales Questionnaire was devised originally to assess
    the attachment of psychiatric patients
    c. interview measures may be better than self-report measures in tapping
    into unconscious attachment strategies *
    d. the Adult Attachment Interview requires little training to administer
    and to score
    e. self-report measures should not be used because of their uncertain
    validity.



    126. Unresolved attachment:
    a. May be seen in individuals whose underlying attachment is
    secure/autonomous *
    b. Is equivalent to fearful attachment in the Bartholomew classification
    c. Is not relevant to the understanding of psychopathology
    d. Is evident purely from the content of discourse on the Adult
    Attachment Interview
    e. Is seen only in women who were sexually abused in childhood.



    127. The decision whether to use psychotropic medication in pregnancy
    should take the following into consideration except:
    a. Neonatal toxicity
    b. Teratogenic risk
    c. Hypertonia in the infant *
    d. Severity of the affective disorder
    e. Risk–benefit assessment



    128. The risks of discontinuing maintenance psychotropic medication in
    pregnancy:
    a. Are negligible
    b. Are not associated with relapse
    c. Are decreased by gradual discontinuation *
    d. Does not include suicidal behaviour
    e. Does not include poor prenatal care



    129. Antidepressants taken during pregnancy:
    a. Can be beneficial in the treatment of stress
    b. Can cause perinatal withdrawal symptoms in the infant *
    c. Have an unacceptably high risk of minor foetal abnormalities
    d. Have a definite association with long-term neurobehavioural problems
    in the child
    e. Are linked with a high incidence of congenital malformations.



    130. The following medications have been shown to be helpful in the
    treatment of alcohol use disorders and depression:
    a. Buspirone
    b. Dothiepin
    c. Paroxetine
    d. Mirtazapine
    e. Imipramine *
    131. As regards dual diagnosis:
    a. There is a dearth of studies in the primary care setting *
    b. The USA has published much less research than the UK
    c. These patients visit general practitioners less frequently than does the
    general population
    d. Scotland and England have the same dual diagnosis policy guidance
    e. There is clear evidence about which models of service work best.


    132. Clozapine has been shown to be useful in the treatment of
    schizophrenia complicated by the use of:
    a. Amphetamines
    b. Methadone
    c. Cannabis *
    d. Cocaine
    e. Heroin


    133. The best way to recognise whether or not a variable is growing
    exponentially over time is by:
    a. Plotting variable against time and looking for a straight-line pattern
    b. Calculating the least squares regression line of the variable against
    time and examining residuals
    c. Plotting logarithm of a variable against time and looking for a straight
    line pattern *
    d. Smoothening the time series by running medians of three or five
    e. Smoothening the scatter plot by median trace

    134. Which of the following is not correct?
    a. The probability of a Type I error is controlled by the selection of alpha
    level
    b. The probability of type II error is controlled by the sample size
    c. The power of a test depends on the sample size and the distance
    between the null and alternate hypothesis
    d. The p value measures the probability that null hypothesis is true *
    e. The rejection region is controlled by the alpha level and alternate
    hypothesis


    135. A study was conducted to investigate the associations of severity
    of depression to suicide. The following information was recorded:
    I. Sex (0 Female, 1 Male)
    II. Age (In Years)
    III. Severity of depression (0 Mild, 1 Moderate, 2 Severe)
    IV. Duration of symptoms in years
    The scale of these variables are:
    a. Nominal, Ratio, Ordinal, Ratio *
    b. Nominal, Ratio, Ordinal, Interval
    c. Ordinal, Interval, Ordinal, Interval
    d. Nominal, Interval, Ratio, Interval
    e. Ordinal, Interval, Nominal, Ratio


    136. In a study about suicide, the study group has decided to increase
    the size of its random sample of attempted patients from 1000 people
    to 3500 people. The effect of this increase is to:
    a. Reduce the bias of the estimate
    b. Increase the standard error of the estimate
    c. Reduce the variability of the estimate *
    d. Increase the confidence interval width for the parameter
    e. Have no effect because the population size is the same


    137. Which of the following is correct?
    a. We do not need to randomize if our sample size is sufficiently large
    b. A large sample size always ensures that our sample is representative
    of the population *
    c. If all other things are equals we need a larger sample size for a larger
    population
    d. In a properly chosen sample, an estimate will be less variable with
    larger sample size and hence it is more precise
    e. In random samples, the randomisation ensures that we get precise and
    accurate estimates


    138. Causal relationship of Dementia and Huntington’s disease can be
    studied most appropriately by?
    a. Randomised control design
    b. Cohort design *
    c. Survey
    d. Qualitative design
    e. Case control design



    139. Correlation coefficient
    a. Is a measure on a scale of 0 to 1
    b. Describes the degree of agreement between two variables
    c. A positive value implies that a rise in one variable accompanies rise in
    the other
    d. Is demonstrated by the symbol k (kappa) *
    e. Spearman’s correlation coefficient is a non-parametric method of
    detecting correlation coefficient*



    140. Which of the following is not a necessary assumption underlying
    the use of Analysis of Variance technique?
    a. The sample are independent and randomly selected
    b. The populations are normally distributed
    c. The variances of the populations are the same
    d. The means of the populations are equal *
    e. All of the above
    Theme: Study Design
    (4 Marks)
    Options:
    A. Case Control study
    B. Cohort Study
    C. Cross Sectional Study
    D. Epidemiological study
    E. Qualitative study
    F. Randomized controlled trial



    From the study described below, select the most appropriate design from
    the list above
    1. To investigate the relationship between alcohol consumption and
    depression, a group of patients attending the psychiatric outpatient clinic
    with a diagnosis of depression were questioned about their alcohol
    consumption. A group of non-depressed age and sex matched patients
    attending the clinic were also questioned about their alcohol consumption
    using an identical protocol. (choose one) A
    2. To investigate the relationship between certain solvents and dementia, all
    employees at a factory were questioned about their exposure to an
    industrial solvent, and the amount and length of exposure measured.
    These subjects were regularly monitored, and after 20 years a copy of the
    death certificate for all those who had died was obtained. (choose one) B
    3. A survey was conducted of a random sample of junior doctors employed at
    a particular hospital. Among other questions, the questionnaire asked
    about the grade of the doctor and whether he/she was satisfied with the
    psychiatric training they were receiving. (choose one) C
    4. A group of patients were invited to a focus group to elicit their views on
    drug treatment and reasons for non-compliance. (choose one) E


    Theme: Calculations in critical appraisal
    (5 Marks)
    Options:
    A. 0
    B. 1
    C. 4
    D. 5
    E. 6
    F. 8
    G. 9
    H. 20
    I. 50
    J. 80
    K. 100
    L. infinity
    For each of the questions below, select the most appropriate number
    from the list above.
    5. The usual upper limit of risk of type II error (expressed as a percentage)
    in power calculations for randomised trials (choose one) H
    6. The odds of an event if it happens 90% of the time. (choose one) G
    7. The ideal Number Needed to Treat. (choose one) B
    8. The value which, if crossed, the 95% confidence interval of a Number
    Needed to Treat (NNT) becomes non-significant. (choose one) L
    9. The sensitivity of a test, expressed as a percentage where 80 people were
    classified “true positive” and 20 people were classified “false
    negative” (choose one) J



    Theme: Neuropsychological Tests
    (4 Marks)
    Options
    A. Cognitive Estimates Test
    B. Digit span
    C. Go – no go test
    D. Mini Mental State Examination
    E. National Adult Reading Test
    F. Raven’s progressive matrices
    G. Rivermead Behavioural Memory Test
    H. Stroop Test
    I. Wechsler Memory Scale
    J. Wisconsin Card Sorting Test
    A 54 year man presents with a year’s history of steadily progressive
    personality change. He has become increasingly apathetic and appears
    depressed but his main complaint is of increasing frontal headaches. On
    examination he has word finding difficulties. EEG shows frontal slowing,
    greater on the left.
    10. You are concerned that he may have an intracranial space occupying
    lesion. Which test would you use to obtain a quick estimate of his current
    performance IQ? (choose one) F
    11. The test indicates that his current performance IQ is in the low average
    range. Which test would enable you to estimate his premorbid IQ, ie. his
    IQ before any brain damage he may have sustained in recent
    months/years? (choose one) E
    12. The estimate of his premorbid IQ is 15 points higher than his current
    performance IQ. It is recommended that he has a full WAIS IQ assessment
    to measure both performance and verbal IQ. On the WAIS his verbal IQ is
    found to be impaired over and above his performance IQ. Which test is
    part of the WAIS verbal subtests? (choose one) B
    13. An MRI scan shows a large meningioma compressing dorsolateral
    prefrontal cortex on the left. Which test result is most likely to be
    impaired? (choose one) J



    Theme: Disorders of Perception
    (4 Marks)
    Options:
    A. Completion illusion
    B. Delusional perception
    C. Dysmegalopsia
    D. Extracampine hallucination
    E. Functional hallucination
    F. Haptic hallucination
    G. Hygric hallucination
    H. Pareidolic illusion
    I. Reflex hallucination
    J. Synaesthesia
    Which of the above descriptive psychopathological terms refers to the
    following symptoms?
    14. “I hear the voice of my long dead father, as if he were talking to me know,
    when I hear water running from the bath tap.” (choose one) E
    15. “I hear the voice of my father speaking to me from the other side of the
    city.” (choose one) D
    16. A young woman describes looking up into the clouds and seeing an image
    of her fiancé. (choose one) H
    17. A 22 year old woman with schizophrenia describes the sensation that
    somebody is touching her body in intimate places. (choose one) F



    Theme: Aetiology
    (4 Marks)
    Options
    A. Alcohol
    B. Childhood sexual abuse
    C. Diabetes
    D. Having given birth 6 months previously
    E. Impaired hearing
    F. Loss of mother before age 14
    G. Peri-natal hypoxia
    H. Reduced visual acuity
    I. Regular use of NSAIDs
    J. Smoking
    Which of the above risk factors is most strongly associated with the
    following:
    18. A young man with mood-incongruent auditory hallucinations, complex
    delusions and social withdrawal (choose one) G
    19. A young woman with mood-congruent delusions, auditory hallucinations
    and depressed mood (choose one) D
    20. An older woman with first onset of both somatic and auditory
    hallucinations and delusions (choose one) E
    21. A cognitively intact older man with visual hallucinations (choose one) H



    Theme: Street names of illicit drugs
    (9 Marks)
    Options
    A. Acid
    B. Angel dust
    C. Cactus
    D. Cold turkey
    E. Snow
    F. Poppers
    G. Dope
    H. Juice
    I. Dynamite
    J. Horse
    K. Herb
    L. Hot shot
    M. Dollies
    N. Uncle

    P. Gear

    Match the above street names with the illicit drugs from the list:
    22. Cannabis (choose one) G
    23. Cocaine (choose one) E
    24. Heroin (choose one) P
    25. Amyl Nitrate (choose one) F
    26. LSD (choose one) A
    27. Methadone (choose one) M
    28. Mescaline (choose one) C
    29. Phencyclidine (choose one) B
    30. Alcohol (choose one) H


    Theme: Relative risk
    (11 Marks)
    Options
    A. NICE has advocated the use of fluoxetine in eating disorders with no
    mention of a risk of suicide.
    B. the MHRA has not warned of a risk of suicide with fluoxetine for any
    indication.
    C. warning about a risk of suicide would cast an unwarranted slur on a
    pharmaceutical company, perhaps even leaving one open to a libel action.
    D. the data are consistent with a 70% reduction in risk of suicidal acts.
    E. the risk is a relative risk rather than an absolute risk.
    F. as the data do not reach statistical significance, some would say there is
    an absence of evidence to indicate a risk.
    G. the data on increased risk are consistent with data from other clinical
    groups showing an increased risk.
    An analysis of the incidence of suicidal acts by patients with bulimia
    nervosa treated with fluoxetine compared with placebo shows a relative
    risk for fluoxetine compared with placebo of 1.5 (95% CI 0.3–6.9).
    31. You should warn your eating disorder patient that there is a small risk of
    clinical worsening and possible suicide, even though: (choose five)
    A,B,D,E,F
    32. If your eating disorder patient takes their own life after starting fluoxetine,
    on balance the data point to a probable role for the treatment even
    though: (choose three) D,E,F
    33. At an inquest on your patient, the company would argue that there is no
    risk from their treatment because: (choose three) A,B,F


    Theme: Outcome measures
    (5 marks)
    Options
    A. Fluoxetine clinical trial Hamilton Rating Scale for Depression (HRSD)
    scores.
    B. Fluoxetine clinical trial Clinical Global Impression scores.
    C. Fluoxetine clinical trial data from the Suicide item (item 3) on the HRSD.
    D. Meta-analytic data from fluoxetine trials showing numbers of lives saved
    from suicide compared with placebo.
    E. Visible signs of worsening clinical condition (a sample of n = 1).
    F. What you are told by your local pharmaceutical company representative.
    G. None of the above
    When balancing the benefits of treatment with fluoxetine against the risk
    of suicide from depression trials in which the risk of a suicidal act is 1.9
    times greater on fluoxetine than on placebo (95% CI 0.2–16.0):
    34. Real-life outcome measures of benefit to set against the real-life hazard
    outcomes of a suicidal act can be found by consulting: (choose one) G
    35. You are likely to be told by some advocates of evidence-based medicine
    that you should pay no heed to data such as ___ because that has not
    been powered sufficiently to give a reliable index of what is happening.
    (choose one) E
    36. The following are termed surrogate outcome measures: (choose three)
    A,B,C



    Theme: Interpretation of results
    (7 Marks)
    Options
    A. Suggest a new trial with a few hundred participants using a rating scale
    sensitive to the emergence of suicidal ideation.
    B. Suggest a new trial with a few hundred participants using a rating scale
    for treatment-emergent agitation.
    C. Suggest re-analysing the data to look for rates of drop out owing to
    agitation.
    D. Suggest a new trial in 50 000 patients.
    E. Suggest looking at national suicide rates since the introduction of these
    drugs.
    F. Conclude that the key thing now is to establish which patients are at risk
    from active treatment.
    G. Conclude that the problem is rare – it only occurs at a rate of 1 per 1000
    treated patients, and psychiatric conditions are linked to a risk of suicide.
    H. Conclude that the problem is real and despite the absolute risk being low,
    given the millions likely to take the treatment the final tally may approach
    the dimensions of a public health disaster.
    You are a researcher given access to data on 15 000 patients, 11 000
    exposed to active treatment and 4000 to placebo, and you have 11
    suicides on active agent and none on placebo for a condition not usually
    linked to suicide.
    37. When consulted by the MHRA, NICE or a company making one of these
    compounds on how to investigate this hazard further, you: (choose four)
    A,B, C, F
    38. If hide the problem, while at the same time trying to appear scientific, you
    might: (choose two) D, E
    39. When asked by the media whether this is a real issue, you: (choose one)
    H



    Theme: Confidence intervals
    (Six marks)
    Options
    A. An arbitrary dichotomisation of the data.
    B. Upper and lower limit values that are equally likely.
    C. The range within which the true value lies.
    D. A modern method to test for statistical significance.
    E. The experiment was not sufficiently powered to achieve a result.
    F. The experiment was not sufficiently discriminating to achieve a result.
    Confidence intervals are consistent with all possible values. You,
    however, are faced with an analysis of the incidence of suicidal acts in
    patients with bulimia nervosa treated with fluoxetine compared with
    placebo that shows a relative risk of suicidal acts on fluoxetine of 1.5
    compared with placebo with a 95% CI of 0.3–6.9.
    40. This interval offers: (choose two) A,B
    41. This interval does not offer: (choose two) C,D
    42. Where the interval ranges broadly and includes 1.0, it suggests: (choose
    one) F
    43. The following suggestion is designed to block rather than assist your
    understanding of the issues: (choose one) E







    Blog EntryMar 16, '08 3:18 PM
    for everyone
    Impairment in:
    Comprehension
    Orientation
    Memory
    Thinking
    Learning
    Language
    Calculation
    Judgement
    Motivation
    Emotional Control
    Social Behaviours
    Activities of Daily Living
    Blog EntryJan 19, '08 4:12 PM
    for everyone
    Pages:1234

    Comments